Разложение в ряд фурье функции: Ряд Фурье онлайн

Содержание

Ряд Фурье. Разложение функции в ряд Фурье. Разложение функции в ряд синусов и косинусов.

Навигация по справочнику TehTab.ru:  главная страница / / Техническая информация/ / Математический справочник/ / Степенные ряды Тейлора, Маклорена (=Макларена) и периодический ряд Фурье. Разложение функций в ряды. / / Ряд Фурье. Разложение функции в ряд Фурье. Разложение функции в ряд синусов и косинусов.

Ряд Фурье. Разложение функции в ряд Фурье. Разложение функции в ряд синусов и косинусов.

Ряд Фурье периодических функций с периодом 2π. Ряд Фурье непериодических функций с периодом 2π.
Четные и нечетные функции. Разложение в ряд Фурье по косинусам.
Разложение в ряд Фурье по синусам.
Ряд Фурье на полупериоде.
Ряд Фурье для произвольного интервала. Ряд Фурье на полупериоде для функций, заданных на интервале L≠2π.

Ряд Фурье периодических функций с периодом 2π.

Ряд Фурье позволяет изучать периодические (непериодические) функции, разлагая их на компоненты. Переменные токи и напряжения, смещения, скорость и ускорение кривошипно-шатунных механизмов и акустические волны — это типичные практические примеры применения периодических функций в инженерных расчетах.

Разложение в ряд Фурье основывается на предположении, что все имеющие практическое значение функции в интервале -π ≤x≤ π можно выразить в виде сходящихся тригонометрических рядов (ряд считается сходящимся, если сходится последовательность частичных сумм, составленных из его членов):

Стандартная (=обычная) запись через сумму sinx и cosx

f(x)=ao+ a1cosx+a2cos2x+a3cos3x+. ..+b1sinx+b2sin2x+b3sin3x+…,

где ao, a1,a2,…,b1,b2,.. — действительные константы, т.е.

(1)

Где для диапазона от -π до π коэффициенты ряда Фурье рассчитываются по формулам:

Коэффициенты ao,an и bn называются коэффициентами Фурье, и если их можно найти, то ряд (1) называется рядом Фурье, соответствующим функции f(x). Для ряда (1) член (a1cosx+b1sinx) называется первой или основной гармоникой,

Другой способ записи ряда — использование соотношения acosx+bsinx=csin(x+α)

f(x)=ao+c1sin(x+α1)+c2sin(2x+α2)+…+cnsin(nx+αn)

Где ao — константа, с 1=(a12+b12)1/2 , с n=(an2+bn2)1/2— амплитуды различных компонент, а фазовый угол равен an=arctg an/bn.

Для ряда (1) член (a1cosx+b1sinx) или c1sin(x+α1) называется первой или основной гармоникой, (a2cos2x+b2sin2x) или c2sin(2x+α2) называется второй гармоникой и так далее.

Для точного представления сложного сигнала обычно требуется бесконечное количество членов. Однако во многих практических задачах достаточно рассмотреть только несколько первых членов.

Ряд Фурье непериодических функций с периодом 2π.

Разложение непериодических функций.

Если функция f(x) непериодическая, значит, она не может быть разложена в ряд Фурье для всех значений х. Однако можно определить ряд Фурье, представляющий функцию в любом диапазоне шириной 2π.

Если задана непериодическая функция, можно составить новую функцию, выбирая значения f(x) в определенном диапазоне и повторяя их вне этого диапазона с интервалом 2π. Поскольку новая функция является периодической с периодом 2π, ее можно разложить в ряд Фурье для всех значений х. Например, функция f(x)=x не является периодической. Однако, если необходимо разложить ее в ряд Фурье на интервале от о до 2π, тогда вне этого интервала строится периодическая функция с периодом 2π (как показано на рис. ниже) .

Для непериодических функций, таких как f(x)=х, сумма ряда Фурье равна значению f(x) во всех точках заданного диапазона, но она не равна f(x) для точек вне диапазона. Для нахождения ряда Фурье непериодической функции в диапазоне 2π используется все таже формула коэффициентов Фурье.

Четные и нечетные функции.

Говорят, функция y=f(x) четная, если f(-x)=f(x) для всех значений х. Графики четных функций всегда симметричны относительно оси у (т.е. являются зеркально отраженными). Два примера четных функций: у=х2 и у=cosx.

Говорят, что функция y=f(x) нечетная, если f(-x)=-f(x) для всех значений х. Графики нечетных функций всегда симметричны относительно начала координат.

Многие функции не являются ни четными, ни нечетными.

Разложение в ряд Фурье по косинусам.

Ряд Фурье четной периодической функции f(x) с периодом 2π содержит только члены с косинусами (т.е. не содержит членов с синусами) и может включать постоянный член. Следовательно,

где коэффициенты ряда Фурье,

Разложение в ряд Фурье по синусам.

Ряд Фурье нечетной периодической функции f(x) с периодом 2π содержит только члены с синусами (т.е. не содержит членов с косинусами).

Следовательно,

где коэффициенты ряда Фурье,

Ряд Фурье на полупериоде.

Если функция определена для диапазона, скажем от 0 до π, а не только от 0 до 2π, ее можно разложить в ряд только по синусам или тольо по косинусам. Полученный ряд Фурье называется рядом Фурье на полупериоде.

Если требуется получить разложение Фурье на полупериоде по косинусам функции f(x) в диапазоне от 0 до π, то необходимо составить четную периодическую функцию. На рис. ниже показана функция f(x)=х, построенная на интервале от х=0 до х=π. Поскольку четная функция симметрична относительно оси f(x), проводим линию АВ, как показано на рис. ниже. Если предположить, что за пределами рассмотренного интервала полученная треугольная форма является периодической с периодом 2π, то итоговый график имеет вид, показ. на рис. ниже. Поскольку требуется получить разложение Фурье по косинусам, как и ранее, вычисляем коэффициенты Фурье ao и an

Если требуется получить разложение Фурье на полупериоде по синусам функции f(x) в диапазоне от 0 до π, то необходимо составить нечетную периодическую функцию. На рис. ниже показана функция f(x)=x, построенная на интервале от от х=0 до х=π.

Поскольку нечетная функция симметрична относительно начала координат, строим линию CD, как показано на рис. Если предположить, что за пределами рассмотренного интервала полученный пилообразный сигнал является периодическим с периодом 2π, то итоговый график имеет вид, показанный на рис. Поскольку требуется получить разложение Фурие на полупериоде по синусам, как и ранее, вычисляем коэффициент Фурье. b

 

Ряд Фурье для произвольного интервала.

Разложение периодической функции с периодом L.

Периодическая функция f(x) повторяется при увеличении х на L, т.е. f(x+L)=f(x). Переход от рассмотренных ранее функций с периодом 2π к функциям с периодом L довольно прост, поскольку его можно осуществить с помощью замены переменной.

Чтобы найти ряд Фурье функции f(x) в диапазоне -L/2≤x≤L/2, введем новую переменную u таким образом, чтобы функция f(x) имела период 2π относительно u. Если u=2πх/L, то х=-L/2 при u=-π и х=L/2 при u=π.

Также пусть f(x)=f(Lu/2π)=F(u). Ряд Фурье F(u) имеет вид

Где коэффициенты ряда Фурье, 

Однако чаще приведенную выше формулу приводят к зависимости от х. Поскольку u=2πх/L, значит, du=(2π/L)dx, а пределы интегрирования — от -L/2 до L/2 вместо — π до π. Следовательно, ряд Фурье для зависимости от х имеет вид

где в диапазоне от -L/2 до L/2 коэффициенты ряда Фурье,

(Пределы интегрирования могут быть заменены на любой интервал длиной L, например, от 0 до L)

Ряд Фурье на полупериоде для функций, заданных в интервале L≠2π.

Для подстановки u=πх/L интервал от х=0 до х=L соответствует интервалу от u=0 до u=π. Следовательно, функцию можно разложить в ряд только по косинусам или только по синусам, т. е. в ряд Фурье на полупериоде.

Разложение по косинусам в диапазоне от 0 до L имеет вид

Дополнительная информация от TehTab.ru:


Ряд Фурье. Разложение функции в ряд Фурье. Разложение функции в ряд синусов и косинусов.

Навигация по справочнику TehTab.ru:  главная страница / / Техническая информация/ / Математический справочник/ / Степенные ряды Тейлора, Маклорена (=Макларена) и периодический ряд Фурье. Разложение функций в ряды. / / Ряд Фурье. Разложение функции в ряд Фурье. Разложение функции в ряд синусов и косинусов.

Ряд Фурье. Разложение функции в ряд Фурье. Разложение функции в ряд синусов и косинусов.

Ряд Фурье периодических функций с периодом 2π. Ряд Фурье непериодических функций с периодом 2π.
Четные и нечетные функции. Разложение в ряд Фурье по косинусам.
Разложение в ряд Фурье по синусам. Ряд Фурье на полупериоде.
Ряд Фурье для произвольного интервала. Ряд Фурье на полупериоде для функций, заданных на интервале L≠2π.

Ряд Фурье периодических функций с периодом 2π.

Ряд Фурье позволяет изучать периодические (непериодические) функции, разлагая их на компоненты. Переменные токи и напряжения, смещения, скорость и ускорение кривошипно-шатунных механизмов и акустические волны — это типичные практические примеры применения периодических функций в инженерных расчетах.

Разложение в ряд Фурье основывается на предположении, что все имеющие практическое значение функции в интервале -π ≤x≤ π можно выразить в виде сходящихся тригонометрических рядов (ряд считается сходящимся, если сходится последовательность частичных сумм, составленных из его членов):

Стандартная (=обычная) запись через сумму sinx и cosx

f(x)=ao+ a1cosx+a2cos2x+a3cos3x+. ..+b1sinx+b2sin2x+b3sin3x+…,

где ao, a1,a2,…,b1,b2,.. — действительные константы, т.е.

(1)

Где для диапазона от -π до π коэффициенты ряда Фурье рассчитываются по формулам:

Коэффициенты ao,an и bn называются коэффициентами Фурье, и если их можно найти, то ряд (1) называется рядом Фурье, соответствующим функции f(x). Для ряда (1) член (a1cosx+b1sinx) называется первой или основной гармоникой,

Другой способ записи ряда — использование соотношения acosx+bsinx=csin(x+α)

f(x)=ao+c1sin(x+α1)+c2sin(2x+α2)+…+cnsin(nx+αn)

Где ao — константа, с 1=(a12+b12)1/2 , с n=(an2+bn2)1/2— амплитуды различных компонент, а фазовый угол равен an=arctg an/bn.

Для ряда (1) член (a1cosx+b1sinx) или c1sin(x+α1) называется первой или основной гармоникой, (a2cos2x+b2sin2x) или c2sin(2x+α2) называется второй гармоникой и так далее.

Для точного представления сложного сигнала обычно требуется бесконечное количество членов. Однако во многих практических задачах достаточно рассмотреть только несколько первых членов.

Ряд Фурье непериодических функций с периодом 2π.

Разложение непериодических функций.

Если функция f(x) непериодическая, значит, она не может быть разложена в ряд Фурье для всех значений х. Однако можно определить ряд Фурье, представляющий функцию в любом диапазоне шириной 2π.

Если задана непериодическая функция, можно составить новую функцию, выбирая значения f(x) в определенном диапазоне и повторяя их вне этого диапазона с интервалом 2π. Поскольку новая функция является периодической с периодом 2π, ее можно разложить в ряд Фурье для всех значений х. Например, функция f(x)=x не является периодической. Однако, если необходимо разложить ее в ряд Фурье на интервале от о до 2π, тогда вне этого интервала строится периодическая функция с периодом 2π (как показано на рис. ниже) .

Для непериодических функций, таких как f(x)=х, сумма ряда Фурье равна значению f(x) во всех точках заданного диапазона, но она не равна f(x) для точек вне диапазона. Для нахождения ряда Фурье непериодической функции в диапазоне 2π используется все таже формула коэффициентов Фурье.

Четные и нечетные функции.

Говорят, функция y=f(x) четная, если f(-x)=f(x) для всех значений х. Графики четных функций всегда симметричны относительно оси у (т.е. являются зеркально отраженными). Два примера четных функций: у=х2 и у=cosx.

Говорят, что функция y=f(x) нечетная, если f(-x)=-f(x) для всех значений х. Графики нечетных функций всегда симметричны относительно начала координат.

Многие функции не являются ни четными, ни нечетными.

Разложение в ряд Фурье по косинусам.

Ряд Фурье четной периодической функции f(x) с периодом 2π содержит только члены с косинусами (т.е. не содержит членов с синусами) и может включать постоянный член. Следовательно,

где коэффициенты ряда Фурье,

Разложение в ряд Фурье по синусам.

Ряд Фурье нечетной периодической функции f(x) с периодом 2π содержит только члены с синусами (т.е. не содержит членов с косинусами).

Следовательно,

где коэффициенты ряда Фурье,

Ряд Фурье на полупериоде.

Если функция определена для диапазона, скажем от 0 до π, а не только от 0 до 2π, ее можно разложить в ряд только по синусам или тольо по косинусам. Полученный ряд Фурье называется рядом Фурье на полупериоде.

Если требуется получить разложение Фурье на полупериоде по косинусам функции f(x) в диапазоне от 0 до π, то необходимо составить четную периодическую функцию. На рис. ниже показана функция f(x)=х, построенная на интервале от х=0 до х=π. Поскольку четная функция симметрична относительно оси f(x), проводим линию АВ, как показано на рис. ниже. Если предположить, что за пределами рассмотренного интервала полученная треугольная форма является периодической с периодом 2π, то итоговый график имеет вид, показ. на рис. ниже. Поскольку требуется получить разложение Фурье по косинусам, как и ранее, вычисляем коэффициенты Фурье ao и an

Если требуется получить разложение Фурье на полупериоде по синусам функции f(x) в диапазоне от 0 до π, то необходимо составить нечетную периодическую функцию. На рис. ниже показана функция f(x)=x, построенная на интервале от от х=0 до х=π. Поскольку нечетная функция симметрична относительно начала координат, строим линию CD, как показано на рис. Если предположить, что за пределами рассмотренного интервала полученный пилообразный сигнал является периодическим с периодом 2π, то итоговый график имеет вид, показанный на рис. Поскольку требуется получить разложение Фурие на полупериоде по синусам, как и ранее, вычисляем коэффициент Фурье. b

 

Ряд Фурье для произвольного интервала.

Разложение периодической функции с периодом L.

Периодическая функция f(x) повторяется при увеличении х на L, т.е. f(x+L)=f(x). Переход от рассмотренных ранее функций с периодом 2π к функциям с периодом L довольно прост, поскольку его можно осуществить с помощью замены переменной.

Чтобы найти ряд Фурье функции f(x) в диапазоне -L/2≤x≤L/2, введем новую переменную u таким образом, чтобы функция f(x) имела период 2π относительно u. Если u=2πх/L, то х=-L/2 при u=-π и х=L/2 при u=π. Также пусть f(x)=f(Lu/2π)=F(u). Ряд Фурье F(u) имеет вид

Где коэффициенты ряда Фурье, 

Однако чаще приведенную выше формулу приводят к зависимости от х. Поскольку u=2πх/L, значит, du=(2π/L)dx, а пределы интегрирования — от -L/2 до L/2 вместо — π до π. Следовательно, ряд Фурье для зависимости от х имеет вид

где в диапазоне от -L/2 до L/2 коэффициенты ряда Фурье,

(Пределы интегрирования могут быть заменены на любой интервал длиной L, например, от 0 до L)

Ряд Фурье на полупериоде для функций, заданных в интервале L≠2π.

Для подстановки u=πх/L интервал от х=0 до х=L соответствует интервалу от u=0 до u=π. Следовательно, функцию можно разложить в ряд только по косинусам или только по синусам, т. е. в ряд Фурье на полупериоде.

Разложение по косинусам в диапазоне от 0 до L имеет вид

Дополнительная информация от TehTab.ru:


Использование методов численного интегрирования при разложении функции в ряд Фурье

Заглавная страница
Избранные статьи
Случайная статья
Познавательные статьи
Новые добавления
Обратная связь

КАТЕГОРИИ:

Археология
Биология
Генетика
География
Информатика
История
Логика
Маркетинг
Математика
Менеджмент
Механика
Педагогика
Религия
Социология
Технологии
Физика
Философия
Финансы
Химия
Экология

ТОП 10 на сайте

Приготовление дезинфицирующих растворов различной концентрации

Техника нижней прямой подачи мяча.

Франко-прусская война (причины и последствия)

Организация работы процедурного кабинета

Смысловое и механическое запоминание, их место и роль в усвоении знаний

Коммуникативные барьеры и пути их преодоления

Обработка изделий медицинского назначения многократного применения

Образцы текста публицистического стиля

Четыре типа изменения баланса

Задачи с ответами для Всероссийской олимпиады по праву



Мы поможем в написании ваших работ!

ЗНАЕТЕ ЛИ ВЫ?

Влияние общества на человека

Приготовление дезинфицирующих растворов различной концентрации

Практические работы по географии для 6 класса

Организация работы процедурного кабинета

Изменения в неживой природе осенью

Уборка процедурного кабинета

Сольфеджио. Все правила по сольфеджио

Балочные системы. Определение реакций опор и моментов защемления

⇐ ПредыдущаяСтр 4 из 4

Разложение функции в ряд Фурье, или гармонический анализ, оказывается нужным во многих чисто практических вопросах машиноведения, электротехники и пр. Но в этих случаях очень редко приходится непосредственно пользоваться формулами Эйлера-Фурье:

для вычисления коэффициентов разложения. Дело в том, что функции, которые нужно подвергнуть гармоническому анализу, обыкновенно задаются таблицей своих значений или графиком. Таким образом, аналитического выражения функции в нашем распоряжении нет; иногда к самому гармоническому анализу прибегают именно для того, чтобы таким путем получить хотя бы приближенное аналитическое выражение для функции. В этих условиях для вычисления коэффициентов Фурье нужно обратиться к приближенным методам. Разумеется, на практике приходится пользоваться лишь немногими первыми членами тригонометрического разложения. Коэффициенты ряда Фурье в большинстве случаев убывают, а с ними быстро падает и влияние далеких гармоник.

Обычно дается (или снимается с графика) ряд равноотстоящих ординат, т.е. ряд значений функции , отвечающих равноотстоящим значениям аргумента x. По этим ординатам коэффициенты можно приближенно вычислить, пользуясь методами изложенными выше. Но вычисления здесь оказываются довольно громоздкими, и для того чтобы упростить и, так сказать, автоматизировать их, придумано много различных приемов, один из которых мы и предлагается.


Можно сделать вывод, что ряды Фурье широко применяются в инженерно-технических расчетах. Они часто встречаются при рассмотрении ряда задач измерительной техники, особенно при исследовании колебательных процессов в измерительных системах, а также при анализе результатов измерений нестационарных параметров. Пример буден приведен при решении нулевого варианта контрольной работы.

Вопросы для самопроверки

1. Запишите формулу средних прямоугольников для вычисления определенного интеграла

Примерный вариант и образец выполнения

Контрольной работы по теме

«Ряды Фурье»

Задача 1. Разложить в ряд Фурье функцию , имеющую период .

Решение. Построим график функции

Эта функция f(x) имеет период , одну точку разрыва первого рода x=0 на отрезке , отрезок можно разбить на два отрезка так, что внутри каждого из них функция f(x) монотонна.

По формуле (2) найдем коэффициент этого ряда.

.

Найдем по формуле (3)

По формуле (4) найдем аналогичным образом

.

Подставляя коэффициенты в формулу (1), получаем или .

Это равенство справедливо во всех точках, кроме точек разрыва. В каждой точке разрыва сумма ряда равна среднему арифметическому ее предельных значений изнутри отрезка, то есть в точке x=0.

= , а на концах отрезка в точках и = .

Ответ.

Построим график S4(x)

 

Задача 2. Разложить в ряд Фурье функцию , заданную на отрезке сначала по синусам, затем по косинусам.

Решение. Построим график

1. Продолжая эту функцию на промежуток нечетным образом, получим функцию, ряд Фурье для которой составлен в §2, пример 2.

Ряд для такого разложения

Построим S5(x)

2. Продолжая эту функцию на промежуток четным образом. Построим график

Эта функция f(x) имеет период , четная, продолжена непрерывно.

. Найдем =

Это равенство справедливо во всех точках числовой прямой.

Построим график S4(x)

Задача3.Разложить в ряд Фурье периодическую функцию , с периодом Т=6.

Решение. Построим эскиз графика функции

Проверив выполнение условий Дирихле для функции, переходим к вычислению коэффициентов Фурье. Заданная функция общего вида с периодом Т=6, l=3, поэтому в разложении ее ряд Фурье имеет вид: .

Подставляя коэффициенты в формулу ряда, получаем или . Это равенство имеет место во всех точках, кроме точек -3 и 3. В каждой из этих точек сумма ряда равна среднему арифметическому ее предельных значений справа и слева, то есть .

Построим график S5 (x)

Можно совместить оба графика на одном чертеже

Отметим близость этих графиков.

Задача4. Разложить в ряд Фурье функцию . Построить график S5(x).

Решение. Будем считать функцию периодической с периодом T=3-1=2, l=1 т.е. , T=2, l=1. Построим эскиз графика этой функции

Ряд Фурье для этой функции будет иметь следующий вид: .

Проверив выполнение условий Дирихле для функции, переходим к вычислению коэффициентов Фурье. .

Подставляя коэффициенты в формулу ряда, получаем или Это равенство имеет место во всех точках, кроме точек 1 и 3. В каждой из этих точек сумма ряда равна среднему арифметическому ее предельных значений справа и слева, то есть .

Построим график S5 (x)

 

Задача 5. Разложить в комплексный ряд Фурье периодическую функцию с периодом , определенную следующим образом: . Построить амплитудно-частотный спектр.

Решение. Будем считать функцию периодической с периодом Т =2. Построим график.

Проверив выполнение условий Дирихле для функции , переходим к вычислению коэффи­циентов Фурье по формуле .

Интеграл, стоящий в правой части последнего равенства, опре­деляется по частям:

;

Если , то полученные формулы не дают результата. Поэтому коэффициент надо вычислить иначе׃ , так как интеграл от нечетной функции по симметричному промежутку равен нулю. Окончательно получим

Это равенство имеет место лишь в точках непрерывности функции . В точках разрыва , где k— любое нечетное число, сумма ряда равна нулю. Построим амплитудно-частотный спектр

Задача 6. Разложить в ряд Фурье по косинусам функцию f(x), графически заданную на промежутке [0; 2] (получить первые 4 гармоники разложения).

Решение. По условию функция – четная, задана на отрезке [0; 2] = [0; l], следовательно, ее график на промежутке [–2; 0] симметричен заданному графику относительно оси ординат и период функции T = 2l =4 (длина промежутка [–2; 2]).

Ряд Фурье для четной периодической функции с периодом 2l имеет вид:

, (1)

где , . (2)

Поскольку вид функции ) неизвестен, для вычисления интегралов используем одну из квадратурных формул – формулу средних прямоугольников:

,

где – середина k-го отрезка разбиения промежутка интегрирования [a; b], k = 1, 2, …, m, h – длина шага разбиения промежутка интегрирования: .

Возьмем m = 10, , т.е. разобъем отрезок [0; 2] на 10 равных частей точками и считаем с графика значения функции в серединах полученных отрезков. Чтобы вычислить коэффициенты a0, a1, a2, a4 для первых 4 гармоник разложения функции в ряд Фурье по формулам (2), построим таблицу значений функции f(x) и в полученных точках:

k xk-1/2 f(xk-1/2)
0,1 0,9 0,89 0,86 0,80
0,3 0,25 0,22 0,15 0,04
0,5 – 0,25 – 0,18 0,18
0,7 – 0,4 – 0,18 0,24 0,40
0,9 – 0,2 – 0,03 0,19 0,09
1,1 0,2 – 0,03 – 0,19 0,09
1,3 0,6 – 0,27 – 0,35 0,59
1,5 0,85 – 0,60 0,60
1,7 0,9 – 0,80 0,53 – 0,14
1,9 – 0,99 0,95 – 0,89
3,85 – 1,97 2,38 1,76

Вычислим коэффициенты ряда a0, a1, a2, a4.

;

 

Подставляем найденные коэффициенты в формулу (1) и получаем аппроксимацию функции частичной суммой ряда s3(x):

 

Для сравнения с функцией f(x) построим на промежутке [0; 2] график заданной функции f(x) и график полученной аппроксимации :

 

Если в аппроксимацию sn(x) включить сумму большего числа гармоник, например, 5, то графики s5(x) и функции f(x) практически совпадают:

 

Ответ: , .

 

Варианты контрольной работы по теме «Ряды Фурье»

Задача 1.

Построить эскиз графика, разложить в ряд Фурье следующие функции, периодические с периодом , определить сумму в точках разрыва. Построить график частичной суммы Фурье для n=4.

Функция Функция

 

Задача 2. Разложить в ряд Фурье функцию , заданную формулой на отрезке , сначала по синусам, затем по косинусам. Построить график и частичных сумм для n=4.

Функция Функция
f(x)=2x-1 f(x)=x-4
f(x)=x2-1 f(x)=x2+2
f(x)=-x-1 f(x)=-x-3
f(x)=x2+1 f(x)=x2-3
f(x)=3x-2 f(x)=0.5x-1

 

Задача3.Разложить в ряд Фурье периодическую функцию , с периодом Т=2l. Построить график частичной суммы при n=5.

 

Функция T Функция T
f(x)=x+4 f(x)=2x+1
f(x)=-x+4 f(x)=2x-1
f(x)=2x+4 f(x)=3x+4
f(x)=x+1 f(x)=3x-2
f(x)=-x+2 f(x)=-3x-1

Задача4Разложить в ряд Фурье функцию . Построить график частичной суммы S4(x).

Функция Функция

Задача 5. Разложить в комплексный ряд Фурье периодическую функцию с периодом , определенную следующим образом: , . Построить амплитудно-частотный спектр.

Функция Функция

Задача 6. Разложить в ряд Фурье функцию f(x), графически заданную на промежутке [0; l] (получить первые гармоники разложения).

 

Разложить в ряд Фурье по синусам функцию f(x), графически заданную на промежутке [0; π] (получить первые 4 гармоники разложения).

 

1. Разложить в ряд Фурье по косинусам функцию f(x), графически заданную на промежутке [0; 1] (получить первые 4 гармоники разложения).

 

 

2. Разложить в ряд Фурье по синусам функцию f(x), графически заданную на промежутке [0; 2] (получить первые 4 гармоники разложения).

 

 

3. Разложить в ряд Фурье по косинусам функцию f(x), графически заданную на промежутке [0; 2,5] (получить первые 4 гармоники разложения).

 

 

4. Разложить в ряд Фурье по синусам функцию f(x), графически заданную на промежутке [0; 3] (получить первые 4 гармоники разложения).

 

 

5. Разложить в ряд Фурье по косинусам функцию f(x), графически заданную на промежутке [0; π] (получить первые 4 гармоники разложения).

 

6. Разложить в ряд Фурье по синусам функцию f(x), графически заданную на промежутке [0; 4] (получить первые 4 гармоники разложения).

 

 

 

7. Разложить в ряд Фурье по косинусам функцию f(x), графически заданную на промежутке [0; 5] (получить первые 4 гармоники разложения).

 

8. Разложить в ряд Фурье по синусам функцию f(x), графически заданную на промежутке [0; π] (получить первые 4 гармоники разложения).

 

9. Разложить в ряд Фурье по синусам функцию f(x), графически заданную на промежутке [0; 1] (получить первые 4 гармоники разложения).

 

Рекомендуемая литература

 

1. Письменный, Д.Т. Конспект лекций по высшей математике. В 2 ч. Ч. 1 / Д.Т. Письменный. –М.: Айрис-пресс, 2003. – 288 с.

2. Щипачев, В.С. Высшая математика: учебник для вузов / В.С. Щипачев.– М.: Высш. шк., 1998.– 479 с.

3.Берман Г.Н. Сборник задач по курсу математического анализа: Уч. пособие.- 22-изд., перераб.- СПб., Изд-во «Профессия», 2005.-432с.

4. Щипачев, В.С. Задачник по высшей математике / В.С. Щипачев.– М.: Высш. шк., 2001.– 304 с.

 

⇐ Предыдущая1234



Читайте также:



Как правильно слушать собеседника

Типичные ошибки при выполнении бросков в баскетболе

Принятие христианства на Руси и его значение

Средства массовой информации США



Последнее изменение этой страницы: 2016-04-19; просмотров: 1596; Нарушение авторского права страницы; Мы поможем в написании вашей работы!

infopedia.su Все материалы представленные на сайте исключительно с целью ознакомления читателями и не преследуют коммерческих целей или нарушение авторских прав. Обратная связь — 161.97.168.212 (0.033 с.)

Разложение в ряд Фурье непериодической функции.

⇐ ПредыдущаяСтр 3 из 5Следующая ⇒

Задача разложения непериодической функции в ряд Фурье,  в принципе не отличается от разложения в ряд Фурье периодической функции.

Допустим, что функция f ( x ) задана на отрезке  и является на этом отрезке  кусочно — монотонной.  Рассмотрим произвольную периодическую кусочно — монотонную функцию  c периодом , совпадающую с функцией f ( x ) на отрезке . То есть можно подобрать отрезок , содержащий отрезок  и раскладывать в ряд Фурье функцию на это отрезке.

 

                                                       y

                                                                   f(x)

 

 

                               a — 2T                a a     b a+2T         a + 4T        x

 

Таким образом, функция  f ( x ) была доопределена.  Полученная  функция  разлагается в ряд Фурье на отрезке , являясь периодической с периодом 2T.  Сумма ряда, составленного для функции , во всех точках отрезка  совпадает с функцией f ( x ), поэтому можно считать, что функция f ( x ) разложена в ряд Фурье на отрезке .

Если функция f ( x ) задана на отрезке, равном  2l, то ее разложение ничем не отличается от разложения в ряд периодической функции с периодом 2l. Если же отрезок, на котором задана функция, меньше, чем  2l, то ее можно продолжить на отрезок [a ; a +2l ], так, чтобы условия разложимости в ряд Фурье сохранялись. Вообще говоря, в этом случае продолжение заданной функции на отрезок (интервал) длиной 2l  может быть произведено бесконечным количеством способов, поэтому суммы получившихся рядов будут различны, но все они будут совпадать с заданной функцией f ( x ) на отрезке .

Пример 4 .  Разложить в ряд Фурье функцию .

Решение. Будем считать функцию периодической с периодом , т.е. . Построим график этой функции

Тогда, ряд Фурье для этой функции будет иметь следующий вид: .

Таким образом .  Построим график S3(x).

Вопросы для самопроверки

1. Запишите ряд Фурье для непериодической функции, заданной на некотором интервале (а, b).

§5. Задача о разложении в ряд Фурье функции, заданной на отрезке [0, π] ([0.l]) по синусам или по косинусам

Этот случай можно свести к предыдущему. Для решения задачи достаточно дополнить определение этой функции для значений  x в проме­жутке  по свободному выбору. Теперь уже  будет оп­ределена на отрезке . Далее поступаем так, как описано в  §3.  В силу того, что мы свободны в выборе вида функции на промежутке , то в результате будут получаться различные ряды Фурье в зависимости от этого выбора. Этот факт может быть использован для получения ряда Фурье функции , содержащего или только косинусы, или только синусы.

Если доопределить данную функцию так, чтобы при , то в результате получится четная функция в промежутке , разложение в ряд Фурье такой функции содержит только косинусы. Коэффициенты разложения можно вычислять по форму­лам: , . Таким образом, заданную на отрезке  функцию  мы разложили по косинусам.

Если доопределить данную функцию так, чтобы  при , то в результате получится нечетная функция, рассматриваемая на промежутке . Разложение в ряд Фурье такой функции содержит только синусы. При этом коэффициенты разложения можно вычислять по формуле: . В этом случае функция , заданная на промежутке , будет разложена по синусам.

Графически это можно представить следующим образом:

Из сказанного следует: заданную на промежутке  функ­цию можно разлагать в ряд  Фурье  как по синусам, так и по ко­синусам.

Замечание 1. Нетрудно заметить, что как в случае разложения неперио­дической функции , определенной на отрезке , так и в случае ее разложения на отрезке  периодическое продолжение заданной функции можно и не осуществлять. На это указывают формулы, из которых определяют­ся коэффициенты Фурье. Но чтобы не сделать ошибок, рекомендуется иметь эскиз графика функции с ее четным или нечетным продолжением на про­межутке  и с последующим периодическим продолжением на всю чи­словую прямую.

Замечание 2. В случае разложения неперио­дической функции , определенной на отрезке  периодическое продолжение производится аналогично функции, определенной на . Формулы, из которых определяют­ся ряд и  коэффициенты Фурье, выбираются соответственно для нечетной функции:  ,

для четной функции.

Пример 5 . Разложить в ряд Фурье функцию    по синусам.

Решение. Продолжив заданную функцию  нечетным образом на промежуток , получим функцию, удовлетворяющую условиям Дирихле на отрезке длиной .

Найдем коэффициенты Фурье для этой функции. Так как она нечетна и, кроме того, симметрична относительно оси абсцисс при совмещении двух полупериодов, ее ряд Фурье содержит только нечетные синусоиды: , .


Окончательно получаем .

Построим график S3(x)

Во многих задачах приходится разлагать в ряд Фурье функцию, заданную на промежутке . При этом функция  на этом промежутке оказывается не только непрерывной, но и дифференцируемой. В этом случае мы можем разложить в ряд Фурье данную функцию,  как по синусам, так и по косинусам. Спрашивается, какому разложению отдать предпочтение? Какой ряд будет обладать лучшими свойствами сходимости? Для практики решение этих вопросов имеет немаловажное значение.

Характер сходимости ряда Фурье определяется свойствами заданной функции в граничных точках  и . Если функция  в этих точках отлична от нуля, то периодическое продолжение ее по принципу нечетной функции приведет к разрыву в двух точкам  и . Эти разрывы легко ликвидируются, если определить  как четную функцию. По этой причине разложение в ряд по косинусам будет обладать гораздо лучшими свойствами сходимости, чем разложение по синусам. В этом случае коэффи­циенты ряда косинусов убывают со  скоростью  а коэффициен­ты ряда синусов — со скоростью .

Если теперь допустить, что  в точках  и  при­нимает значения, равные нулю, то разложение в ряд по синусам дает гораздо лучшую сходимость, чем разложение в ряд по коси­нусам, так как периодическое продолжение функции  по принципу нечетной функции обеспечивает непрерывность и самой функции и ее первой производной, в то время как периодическое продолжение по принципу четной функ­ции приводит к разрыву первой производной в точках  и . В этом случае  коэффициенты ряда синусов убывают со скоростью ,  подходящей для многих приложений рядов Фурье.

Вопросы для самопроверки

1. Запишите ряд Фурье для разложения  функции, заданной  на промежутке , по синусам и косинусам

⇐ Предыдущая12345Следующая ⇒



Разложить график в ряд фурье.

Ряд Фурье

Разложение в ряд Фурье четных и нечетных функций разложение функции заданной на отрезке в ряд по синусам или по косинусам Ряд Фурье для функции с произвольным периодом Комплексная запись ряда Фурье Ряды Фурье по общим ортогональным системам функций Ряд Фурье по ортогональной системе Минимальное свойство коэффициентов Фурье Неравенство Бесселя Равенство Парсеваля Замкнутые системы Полнота и замкнутость систем


Разложение в ряд Фурье четных и нечетных функций Функция f(x), определенная на отрезке \-1, где I > 0, называется четной, если График четной функции симметричен относительно оси ординат. Функция f(x), определенная на отрезке J), где I > 0, называется нечетной, если График нечетной функции симметричен относительно начала координат. Пример. а) Функция является четной на отрезке |-jt, jt), так как для всех х е б) Функция является нечетной, так как Разложение в ряд Фурье четных и нечетных функций разложение функции заданной на отрезке в ряд по синусам или по косинусам Ряд Фурье для функции с произвольным периодом Комплексная запись ряда Фурье Ряды Фурье по общим ортогональным системам функций Ряд Фурье по ортогональной системе Минимальное свойство коэффициентов Фурье Неравенство Бесселя Равенство Парсеваля Замкнутые системы Полнота и замкнутость систем в) Функция f(x)=x2-x, где не принадлежит ни к четным, ни к нечетным функциям, так как Пусть функция f(x), удовлетворяющая условиям теоремы 1, является четной на отрезке х|. п функцию 4 Так как эта функция четная и удовлетворяет условиям теоремы 1, то ее ряд Фурье имеет вид Находим коэффициенты Фурье. Имеем Применяя дважды интегрирование по частям, получим, что Значит, ряд Фурье данной функции выглядит так: или, в развернутом виде, Это равенство справедливо для любого х € , так как в точках х = ±ir сумма ряда совпадает со значениями функции f(x) = х2, поскольку Графики функции f(x) = х и суммы полученного ряда даны на рис. Замечание. Этот ряд Фурье позволяет найти сумму одного из сходящихся числовых рядов, а именно, при х = 0 получаем, что Пример 2. Разложить в ряд Фурье на интервале функцию /(х) = х. Функция /(х) удовлетворяет условиям теоремы 1, следовательно ее можно разложить в ряд Фурье, который в силу нечетности этой функции будет иметь вид Интегрируя по частям, находим коэффициенты Фурье Следовательно, ряд Фурье данной функции имеет вид Это равенство имеет место для всех х В точках х — ±тг сумма ряда Фурье не совпадает со значениями функции /(х) = х, так как она равна Вне отрезка [-*, я-] сумма ряда является периодическим продолжением функции /(х) = х; ее график изображен на рис. 6. § 6. Разложение функции, заданной на отрезке, в ряд по синусам или по косинусам Пусть ограниченная кусочно-монотонная функция / задана на отрезке . Значения этой функции на отрезке 0| можно доопределить различным образом. Например, можно определить функцию / на отрезке тс] так, чтобы /. В этом случае говорят, что) «продолжена на отрезок 0] четным образом»; ее ряд Фурье будет содержать только косинусы. Если же функцию /(ж) определить на отрезке [-л-, тс] так, чтобы /(, то получится нечетная функция, и тогда говорят, что / «продолжена на отрезок [-*, 0] нечетным образом»; в этом случае се ряд Фурье будет содержать только синусы. Итак, каждую ограниченную кусочно-монотонную функцию /(ж), определенную на отрезке , можно разложить в ряд Фурье и по синусам, и по косинусам. Пример 1. Функцию разложить в ряд Фурье: а) по косинусам; б) по синусам. М Данная функция при ее четном и нечетном продолжениях в отрезок |-х,0) будет ограниченной и кусочно-монотонной. а) Продолжим /(z) в отрезок 0) а) Продолжим j\x) в отрезок (-тг,0| четным образом (рис. tj будет периодической функцией аргумента t с периодом и ее можно разложить на отрезке в ряд Фурье Возвращаясь к переменной ж, т. е. положив, получим Все теоремы, справедливые для рядов Фурье периодических функций с периодом 2тг, остаются в силе и для периодических функций с произвольным периодом 21. В частности, сохраняет свою силу и достаточный признак разложимости функции в ряд Фурье. Пример 1. Разложить в ряд Фурье периодическую функцию с периодом 21, заданную на отрезке [-/,/] формулой (рис.9). Так как данная функция четная, то ее ряд Фурье имеет вид Подставляя в ряд Фурье найденные значения коэффициентов Фурье, получим Отметим одно важное свойство периодических функций. Теорема 5. Если функция имеет период Т и интегрируема, то для любого числа а выполняется равенство m. е. интеграл no отрезку, длина которого равна периоду Т, имеет одно и то же значение независимо от положения этого отрезка на числовой оси. В самом деле, Делаем замену переменной во втором интеграле, полагая. Это дает и следовательно, Геометрически это свойство означает, что в случае площади заштрихованных на рис. 10 областей равны между собой. В частности, для функции f(x) с периодом получим при Разложение в ряд Фурье четных и нечетных функций разложение функции заданной на отрезке в ряд по синусам или по косинусам Ряд Фурье для функции с произвольным периодом Комплексная запись ряда Фурье Ряды Фурье по общим ортогональным системам функций Ряд Фурье по ортогональной системе Минимальное свойство коэффициентов Фурье Неравенство Бесселя Равенство Парсеваля Замкнутые системы Полнота и замкнутость систем Пример 2. Функция x является периодической с периодом В силу нечетности данной функции без вычисления интегралов можно утверждать, что при любом Доказанное свойство, в частности, показывает, что коэффициенты Фурье периодической функции f(x) с периодом 21 можно вычислять по формулам где а — произвольное действительное число (отметим, что функции cos — и sin имеют период 2/). Пример 3. Разложить в ряд Фурье заданную на интервале функцию с периодом 2х (рис. 11). 4 Найдем коэффициенты Фурье данной функции. Положив в формулах найдем, что для Следовательно, ряд Фурье будет выглядеть так: В точке х = jt (точка разрыва первого рода) имеем §8. Комплексная запись ряда Фурье В этом параграфе используются некоторые элементы комплексного анализа (см. главу XXX, где все, производимые здесь действия с комплексными выражениями, строго обоснованы). Пусть функция f(x) удовлетворяет достаточным условиям разложимости в ряд Фурье. Тогда на отрезке ж] ее можно представить рядом вида Используя формулы Эйлера Подставляя эти выражения в ряд (1) вместо cos пх и sin пху будем иметь Введем следующие обозначения Тогда ряд (2) примет вид Таким образом, ряд Фурье (1) представлен в комплексной форме (3). Найдем выражения коэффициентов через интегралы. Имеем Аналогично находим Окончательно формулы для с„, с_п и со можно записать так: . . Коэффициенты с„ называются комплексными коэффициентами Фурье функции Для периодической функции с периодом) комплексная форма ряда Фурье примет вид где коэффициенты Сп вычисляются по формулам Сходимость рядов (3) и (4) понимается так: ряды (3) и (4) называются сходящимися для данного значения ж, если существуют пределы Пример. Разложить в комплексный ряд Фурье функцию периода Данная функция удовлетворяет достаточным условиям разложимости в ряд Фурье. Пусть Найдем комплексные коэффициенты Фурье этой функции. Имеем для нечетных для четных n, или,короче. Подставляя значения), окончательно получим Заметим, что этот ряд можно записать и так: Ряды Фурье по общим ортогональным системам функций 9.1. Ортогональные системы функций Обозначим через множество всех (действительных) функций, определенных и интегрируемых на отрезке [а, 6] с квадратом, т. е. таких, для которых существует интеграл В частности, все функции f(x), непрерывные на отрезке [а, 6], принадлежат 6], и значения их интегралов Лебега совпадают со значениями интегралов Римана. Определение. Система функций, где, называется ортогональной на отрезке [а, Ь\, если Условие (1) предполагает, в частности, что ни одна из функций не равна тождественно нулю. Интеграл понимается в смысле Лебега. и назовем величину нормой функции Если в ортогональной системе для всякого п имеем, то система функций называется ортонормированной. Если система {у>„(ж)} ортогональна, то система Пример 1. Тригонометрическая система ортогональна на отрезке. Система функций является ортонормированной системой функций на, Пример 2. Косинус-система и синус-система ортонормирована. Введем обозначение являются ортогональными на отрезке (0, f|, но не ортонормированными (при I Ф- 2). так как их нормы COS Пример 3. Многочлены, определяемые равенством, называются многочленами (полиномами) Лежандра. При п = 0 имеем Можно доказать, что функции образуют ортонормированную систему функций на отрезке. Покажем, например, ортогональность полиномов Лежандра. Пусть т > п. В этом случае, интегрируя п раз по частям, находим поскольку для функции t/m = (z2 — I)m все производные до порядка m — I включительно обращаются в нуль на концах отрезка [-1,1). Определение. Система функций {pn(x)} называется ортогональной на интервале (а, Ь) свесом р(х), если: 1) для всех п = 1,2,… существуют интегралы Здесь предполагается, что весовая функция р(х) определена и положительна всюду на интервале (а, Ь) за возможным исключением конечного числа точек, где р(х) может обращаться в нуль. п(я)}- Числа Сп называются коэффициентами Фурье функции f(x) по этой системе. Знак ~ в формуле (6) означает лишь, что числа Сп связаны с функцией /(ж) формулой (5) (при этом не предполагается, что ряд справа вообще сходится, а тем более сходится к функции f(x)). Поэтому естественно возникает вопрос: каковы свойства этого ряда? В каком смысле он «представляет» функцию f(x)? 9.3. Сходимость в среднем Определение. Последовательность, сходится к элементу ] в среднем, если норма в пространстве Теорема 6. Если последовательность } сходится равномерно, то она сходится и в среднем. М Пусть последовательность {)} сходится равномерно на отрезке [а, Ь] к функции /(х). Это означает, что для всякого при всех достаточно больших п имеем Следовательно, откуда вытекает наше утверждение. Обратное утверждение неверно: последовательность {} может сходиться в среднем к /(х), но не быть равномерно сходящейся. Пример. Рассмотрим последовательность пх Легко видеть, что Но эта сходимость не равномерна: существует е, например, такое, что сколь бы большим ни было л, на отрезке , Разложение в ряд Фурье четных и нечетных функций разложение функции заданной на отрезке в ряд по синусам или по косинусам Ряд Фурье для функции с произвольным периодом Комплексная запись ряда Фурье Ряды Фурье по общим ортогональным системам функций Ряд Фурье по ортогональной системе Минимальное свойство коэффициентов Фурье Неравенство Бесселя Равенство Парсеваля Замкнутые системы Полнота и замкнутость систем и пусть Обозначим через с* коэффициенты Фурье функции /(х) по ортонормированной системе ь Рассмотрим линейную комбинацию где n ^ 1 — фиксированное целое число, и найдем значения постоянных, при которых интеграл принимает минимальное значение. Запишем его подробнее Интефируя почленно, в силу ортонормированности системы получим Первые два слагаемых в правой части равенства (7) не зависят, а третье слагаемое неотрицательно. Поэтому интеграл (*) принимает минимальное значение при ак = ск Интеграл называют средним квадратичным приближением функции /(х) линейной комбинацией Тп(х). Таким образом, среднее квадратичное приближение функции/\ принимает минимальное значение, когда. когда Тп(х) есть 71-я частичная сумма ряда Фурье функции /(х) по системе {. Полагая ак = ск, из (7) получаем Равенство (9) называется тождеством Бесселя. Так как его левая часть неотрицательна, то из него следует неравенство Бесселя Поскольку я здесь произвольно, то неравенство Бесселя можно представить в усиленной форме т. е. для всякой функции / ряд из квадратов коэффициентов Фурье этой функции по ортонормированной системе } сходится. Так как система ортонормирована на отрезке [-х, тг], то неравенство (10) в переводе на привычную запись тригонометрического ряда Фурье дает соотношение do справедливое для любой функции /(х) с интегрируемым квадратом. „(х)} знак неравенства в формуле (10) может быть заменен (для всех функций /(х) 6 Ч) знаком равенства. Получаемое равенство называется равенством Парсеваля-Стеклова (условием полноты). Тождество Бесселя (9) позволяет записать условие (12) в равносильной форме Тем самым выполнение условия полноты означает, что частичные суммы Sn(x) ряда Фурье функции /(х) сходятся к функции /(х) в среднем, т.е. по норме пространства 6]. Определение. Ортонормированная система { называется полной в Ь2[ау Ь], если всякую функцию можно с любой точностью приблизить в среднем линейной комбинацией вида с достаточно большим числом слагаемых, т. е. если для всякой функции/(х) € Ь2[а, Ь\ и для любого е > 0 найдется натуральное число nq и числа а\, а2у…, такие, что No Из приведенных рассуждений следует Теорема 7. Если ортонормированием система } полна в пространстве ряд Фурье всякой функции / по этой системе сходится к f(x) в среднем, т. е. по норме Можно показать, что тригонометрическая система полна в пространстве, Отсюда следует утверждение. Теорема 8. Если функция /о ее тригонометрический ряд Фурье сходится к ней в среднем. 9.5. Замкнутые системы. Полнота и замкнутость систем Определение. Ортонормированная система функций \, называется замкнутой, если в пространстве Li\a, Ь) не существует отличной от нуля функции, ортогональной ко всем функциям В пространстве L2\a, Ь\ понятия полноты и замкнутости ортонормированных систем совпадают. Упражнения 1. Разложите в ряд Фурье в интервале (-я-, ж) функцию 2. Разложите в ряд Фурье в интервале (-тг, тг) функцию 3. Разложите в ряд Фурье в интервале (-тг, тг) функцию 4. Разложите в ряд Фурье в интервале (-jt, тг) функцию 5. Разложите в ряд Фурье в интервале (-тг, тг) функцию f(x) = ж + х. 6. Разложите в ряд Фурье в интервале (-jt, тг) функцию п 7. Разложите в ряд Фурье в интервале (-тг, ж) функцию /(х) = sin2 х. 8. Разложите в ряд Фурье в интервале (-тг, jt) функцию f(x) = у 9. Разложите в ряд Фурье в интервале (-тт, -к) функцию /(х) = | sin х|. 10. Разложите в ряд Фурье в интервале (-я-, тг) функцию /(х) = §. 11. Разложите в ряд Фурье в интервале (-тг, тг) функцию f(x) = sin §. 12. Разложите в ряд Фурье функцию f(x) = п -2х, заданную в интервале (0, х), продолжив ее в интервал (-х, 0): а) четным образом; б) нечетным образом. 13. Разложите в ряд Фурье по синусам функцию /(х) = х2, заданную в интервале (0, х). 14. Разложите в ряд Фурье функцию /(х) = 3-х, заданную в интервале (-2,2). 15. Разложите в ряд Фурье функцию f(x) = |х|, заданную в интервале (-1,1). 16. Разложите в ряд Фурье по синусам функцию f(x) = 2х, заданную в интервале (0,1).

Ряды Фурье — это представление произвольно взятой функции с конкретным периодом в виде ряда. В общем виде данное решение называют разложением элемента по ортогональному базису. Разложение функций в ряд Фурье является довольно мощным инструментарием при решении разнообразных задач благодаря свойствам данного преобразования при интегрировании, дифференцировании, а также сдвиге выражения по аргументу и свертке.

Человек, не знакомый с высшей математикой, а также с трудами французского ученого Фурье, скорее всего, не поймет, что это за «ряды» и для чего они нужны. А между тем данное преобразование довольно плотно вошло в нашу жизнь. Им пользуются не только математики, но и физики, химики, медики, астрономы, сейсмологи, океанографы и многие другие. Давайте и мы поближе познакомимся с трудами великого французского ученого, сделавшего открытие, опередившее свое время.

Человек и преобразование Фурье

Ряды Фурье являются одним из методов (наряду с анализом и другими) Данный процесс происходит каждый раз, когда человек слышит какой-либо звук. Наше ухо в автоматическом режиме производит преобразование элементарных частиц в упругой среде раскладываются в ряды (по спектру) последовательных значений уровня громкости для тонов разной высоты. Далее мозг превращает эти данные в привычные для нас звуки. Все это происходит помимо нашего желания или сознания, само по себе, а вот для того чтобы понять эти процессы, понадобится несколько лет изучать высшую математику.

Подробнее о преобразовании Фурье

Преобразование Фурье можно проводить аналитическими, числительными и другими методами. Ряды Фурье относятся к числительному способу разложения любых колебательных процессов — от океанских приливов и световых волн до циклов солнечной (и других астрономических объектов) активности. Используя эти математические приемы, можно разбирать функции, представляя любые колебательные процессы в качестве ряда синусоидальных составляющих, которые переходят от минимума к максимуму и обратно. Преобразование Фурье является функцией, описывающей фазу и амплитуду синусоид, соответствующих определенной частоте. Данный процесс можно использовать для решения весьма сложных уравнений, которые описывают динамические процессы, возникающие под действием тепловой, световой или электрической энергии. Также ряды Фурье позволяют выделять постоянные составляющие в сложных колебательных сигналах, благодаря чему стало возможным правильно интерпретировать полученные экспериментальные наблюдения в медицине, химии и астрономии.

Историческая справка

Отцом-основателем этой теории является французский математик Жан Батист Жозеф Фурье. Его именем впоследствии и было названо данное преобразование. Изначально ученый применил свой метод для изучения и объяснения механизмов теплопроводности — распространения тепла в твердых телах. Фурье предположил, что изначальное нерегулярное распределение можно разложить на простейшие синусоиды, каждая из которых будет иметь свой температурный минимум и максимум, а также свою фазу. При этом каждая такая компонента будет измеряться от минимума к максимуму и обратно. Математическая функция, которая описывает верхние и нижние пики кривой, а также фазу каждой из гармоник, назвали преобразованием Фурье от выражения распределения температуры. Автор теории свел общую функцию распределения, которая трудно поддается математическому описанию, к весьма удобному в обращении ряду косинуса и синуса, в сумме дающих исходное распределение.

Принцип преобразования и взгляды современников

Современники ученого — ведущие математики начала девятнадцатого века — не приняли данную теорию. Основным возражением послужило утверждение Фурье о том, что разрывную функцию, описывающую прямую линию или разрывающуюся кривую, можно представить в виде суммы синусоидальных выражений, которые являются непрерывными. В качестве примера можно рассмотреть «ступеньку» Хевисайда: ее значение равно нулю слева от разрыва и единице справа. Данная функция описывает зависимость электрического тока от временной переменной при замыкании цепи. Современники теории на тот момент никогда не сталкивались с подобной ситуацией, когда разрывное выражение описывалось бы комбинацией непрерывных, обычных функций, таких как экспонента, синусоида, линейная или квадратичная.

Что смущало французских математиков в теории Фурье?

Ведь если математик был в прав в своих утверждениях, то, суммируя бесконечный тригонометрический ряд Фурье, можно получить точное представление ступенчатого выражения даже в том случае, если оно имеет множество подобных ступеней. В начале девятнадцатого века подобное утверждение казалось абсурдным. Но несмотря на все сомнения, многие математики расширили сферу изучения данного феномена, выведя его за пределы исследований теплопроводности. Однако большинство ученых продолжали мучиться вопросом: «Может ли сумма синусоидального ряда сходиться к точному значению разрывной функции?»

Сходимость рядов Фурье: пример

Вопрос о сходимости поднимается всякий раз при необходимости суммирования бесконечных рядов чисел. Для понимания данного феномена рассмотрим классический пример. Сможете ли вы когда-либо достигнуть стены, если каждый последующий шаг будет вдвое меньше предыдущего? Предположим, что вы находитесь в двух метрах от цели, первый же шаг приближает к отметке на половине пути, следующий — к отметке в три четверти, а после пятого вы преодолеете почти 97 процентов пути. Однако сколько бы вы шагов ни сделали, намеченной цели вы не достигните в строгом математическом смысле. Используя числовые расчеты, можно доказать, что в конце концов можно приблизиться на сколь угодно малое заданное расстояние. Данное доказательство является эквивалентным демонстрации того, что суммарное значение одной второй, одной четвертой и т. д. будет стремиться к единице.

Вопрос сходимости: второе пришествие, или Прибор лорда Кельвина

Повторно данный вопрос поднялся в конце девятнадцатого века, когда ряды Фурье попробовали применить для предсказания интенсивности отливов и приливов. В это время лордом Кельвином был изобретен прибор, представляющий собой аналоговое вычислительное устройство, которое позволяло морякам военного и торгового флота отслеживать это природное явление. Данный механизм определял наборы фаз и амплитуд по таблице высоты приливов и соответствующих им временных моментов, тщательно замеренных в данной гавани в течение года. Каждый параметр представлял собой синусоидальную компоненту выражения высоты прилива и являлся одной из регулярных составляющих. Результаты измерений вводились в вычислительный прибор лорда Кельвина, синтезирующий кривую, которая предсказывала высоту воды как временную функцию на следующий год. Очень скоро подобные кривые были составлены для всех гаваней мира.

А если процесс будет нарушен разрывной функцией?

В то время представлялось очевидным, что прибор, предсказывающий приливную волну, с большим количеством элементов счета может вычислить большое количество фаз и амплитуд и так обеспечить более точные предсказания. Тем не менее оказалось, что данная закономерность не соблюдается в тех случаях, когда приливное выражение, которое следует синтезировать, содержало резкий скачок, то есть являлось разрывным. В том случае, если в устройство вводятся данные из таблицы временных моментов, то оно производит вычисления нескольких коэффициентов Фурье. Исходная функция восстанавливается благодаря синусоидальным компонентам (в соответствии с найденными коэффициентами). Расхождение между исходным и восстановленным выражением можно измерять в любой точке. При проведении повторных вычислений и сравнений видно, что значение наибольшей ошибки не уменьшается. Однако они локализируются в области, соответствующей точке разрыва, а в любой иной точке стремятся к нулю. В 1899 году этот результат был теоретически подтвержден Джошуа Уиллардом Гиббсом из Йельского университета.

Сходимость рядов Фурье и развитие математики в целом

Анализ Фурье неприменим к выражениям, содержащим бесконечное количество всплесков на определенном интервале. В общем и целом ряды Фурье, если изначальная функция представлена результатом реального физического измерения, всегда сходятся. Вопросы сходимости данного процесса для конкретных классов функций привели к появлению новых разделов в математике, например теории обобщенных функций. Она связана с такими именами, как Л. Шварц, Дж. Микусинский и Дж. Темпл. В рамках данной теории была создана четкая и точная теоретическая основа под такие выражения, как дельта-функция Дирака (она описывает область единой площади, сконцентрированной в бесконечно малой окрестности точки) и «ступень» Хевисайда. Благодаря этой работе ряды Фурье стали применимы для решения уравнений и задач, в которых фигурируют интуитивные понятия: точечный заряд, точечная масса, магнитные диполи, а также сосредоточенная нагрузка на балке.

Метод Фурье

Ряды Фурье, в соответствии с принципами интерференции, начинаются с разложения сложных форм на более простые. Например, изменение теплового потока объясняется его прохождением сквозь различные препятствия из теплоизолирующего материала неправильной формы или изменением поверхности земли — землетрясением, изменением орбиты небесного тела — влиянием планет. Как правило, подобные уравнения, описывающие простые классические системы, элементарно решаются для каждой отдельной волны. Фурье показал, что простые решения также можно суммировать для получения решения более сложных задач. Выражаясь языком математики, ряды Фурье — это методика представления выражения суммой гармоник — косинусоид и синусоид. Поэтому данный анализ известен также под именем «гармонический анализ».

Ряд Фурье — идеальная методика до «компьютерной эпохи»

До создания компьютерной техники методика Фурье являлась лучшим оружием в арсенале ученых при работе с волновой природой нашего мира. Ряд Фурье в комплексной форме позволяет решать не только простые задачи, которые поддаются прямому применению законов механики Ньютона, но и фундаментальные уравнения. Большинство открытий ньютоновской науки девятнадцатого века стали возможны только благодаря методике Фурье.

Ряды Фурье сегодня

С развитием компьютеров преобразования Фурье поднялись на качественно новый уровень. Данная методика прочно закрепилась практически во всех сферах науки и техники. В качестве примера можно привести цифровой аудио- и видеосигнал. Его реализация стала возможной только благодаря теории, разработанной французским математиком в начале девятнадцатого века. Так, ряд Фурье в комплексной форме позволил совершить прорыв в изучении космического пространства. Кроме того, это повлияло на изучение физики полупроводниковых материалов и плазмы, микроволновой акустики, океанографии, радиолокации, сейсмологии.

Тригонометрический ряд Фурье

В математике ряд Фурье является способом представления произвольных сложных функций суммой более простых. В общих случаях количество таких выражений может быть бесконечным. При этом чем больше их число учитывается при расчете, тем точнее получается конечный результат. Чаще всего в качестве простейших используют тригонометрические функции косинуса или синуса. В таком случае ряды Фурье называют тригонометрическими, а решение таких выражений — разложением гармоники. Этот метод играет важную роль в математике. Прежде всего, тригонометрический ряд дает средства для изображения, а также изучения функций, он является основным аппаратом теории. Кроме того, он позволяет решать ряд задач математической физики. Наконец, данная теория способствовала развитию вызвала к жизни целый ряд весьма важных разделов математической науки (теорию интегралов, теорию периодических функций). Кроме того, послужила отправным пунктом для развития следующих функций действительного переменного, а также положила начало гармоническому анализу.

Которые уже порядком поднадоели. И я чувствую, что настал момент, когда из стратегических запасов теории пора извлечь новые консервы. Нельзя ли разложить функцию в ряд как-нибудь по-другому? Например, выразить отрезок прямой линии через синусы и косинусы? Кажется невероятным, но такие, казалось бы, далекие друг от друга функции поддаются
«воссоединению». Помимо примелькавшихся степеней в теории и практике существуют и другие подходы к разложению функции в ряд.

На данном уроке мы познакомимся с тригонометрическим рядом Фурье, коснёмся вопроса его сходимости и суммы и, конечно же, разберём многочисленные примеры на разложение функций в ряд Фурье. Искренне хотелось назвать статью «Ряды Фурье для чайников», но это было бы лукавством, поскольку для решения задач потребуются знания других разделов математического анализа и некоторый практический опыт. Поэтому преамбула будет напоминать подготовку космонавтов =)

Во-первых, к изучению материалов страницы следует подойти в отличной форме. Выспавшимися, отдохнувшими и трезвыми. Без сильных эмоций по поводу сломанной лапы хомячка и навязчивых мыслей о тяготах жизни аквариумных рыбок. Ряд Фурье не сложен с точки зрения понимания, однако практические задания требуют просто повышенной концентрации внимания – в идеале следует полностью отрешиться от внешних раздражителей. Ситуация усугубляется тем, что не существует лёгкого способа проверки решения и ответа. Таким образом, если ваше самочувствие ниже среднего, то лучше заняться чем-нибудь попроще. Правда.

Во-вторых, перед полётом в космос необходимо изучить приборную панель космического корабля. Начнём со значений функций, которые должны щёлкаться на автомате:

При любом натуральном значении :

1) . И в самом деле, синусоида «прошивает» ось абсцисс через каждое «пи»:
. В случае отрицательных значений аргумента результат, само собой, будет таким же: .

2) . А вот это знали не все. Косинус «пи эн» представляет собой эквивалент «мигалки»:

Отрицательный аргумент дела не меняет: .

Пожалуй, достаточно.

И, в-третьих, уважаемый отряд космонавтов, необходимо уметь… интегрировать .
В частности, уверенно подводить функцию под знак дифференциала , интегрировать по частям и быть в ладах с формулой Ньютона-Лейбница . Начнём важные предполётные упражнения. Категорически не рекомендую пропускать, чтобы потом не плющило в невесомости:

Пример 1

Вычислить определённые интегралы

где принимает натуральные значения.

Решение : интегрирование проводится по переменной «икс» и на данном этапе дискретная переменная «эн» считается константой. Во всех интегралах подводим функцию под знак дифференциала :

Короткая версия решения, к которой хорошо бы пристреляться, выглядит так:

Привыкаем:

Четыре оставшихся пункта самостоятельно. Постарайтесь добросовестно отнестись к заданию и оформить интегралы коротким способом. Образцы решений в конце урока.

После КАЧЕСТВЕННОГО выполнения упражнений надеваем скафандры
и готовимся к старту!

Разложение функции в ряд Фурье на промежутке

Рассмотрим некоторую функцию , которая определена по крайне мере на промежутке (а, возможно, и на бОльшем промежутке). Если данная функция интегрируема на отрезке , то её можно разложить в тригонометрический ряд Фурье :
, где – так называемые коэффициенты Фурье .

При этом число называют периодом разложения , а число – полупериодом разложения .

Очевидно, что в общем случае ряд Фурье состоит из синусов и косинусов:

Действительно, распишем его подробно:

Нулевой член ряда принято записывать в виде .

Коэффициенты Фурье рассчитываются по следующим формулам:

Прекрасно понимаю, что начинающим изучать тему пока малопонятны новые термины: период разложения , полупериод , коэффициенты Фурье и др. Без паники, это не сравнимо с волнением перед выходом в открытый космос. Во всём разберёмся в ближайшем примере, перед выполнением которого логично задаться насущными практическими вопросами:

Что нужно сделать в нижеследующих заданиях?

Разложить функцию в ряд Фурье. Дополнительно нередко требуется изобразить график функции , график суммы ряда , частичной суммы и в случае изощрённых профессорский фантазий – сделать что-нибудь ещё.

Как разложить функцию в ряд Фурье?

По существу, нужно найти коэффициенты Фурье , то есть, составить и вычислить три определённых интеграла .

Пожалуйста, перепишите общий вид ряда Фурье и три рабочие формулы к себе в тетрадь. Я очень рад, что у некоторых посетителей сайта прямо на моих глазах осуществляется детская мечта стать космонавтом =)

Пример 2

Разложить функцию в ряд Фурье на промежутке . Построить график , график суммы ряда и частичной суммы .

Решение : первая часть задания состоит в разложении функции в ряд Фурье.

Начало стандартное, обязательно записываем, что:

В данной задаче период разложения , полупериод .

Разложим функцию в ряд Фурье на промежутке :

Используя соответствующие формулы, найдём коэффициенты Фурье . Теперь нужно составить и вычислить три определённых интеграла . Для удобства я буду нумеровать пункты:

1) Первый интеграл самый простой, однако и он уже требует глаз да глаз:

2) Используем вторую формулу:

Данный интеграл хорошо знаком и берётся он по частям :

При нахождении использован метод подведения функции под знак дифференциала .

В рассматриваемом задании сподручнее сразу использовать формулу интегрирования по частям в определённом интеграле :

Пара технических замечаний. Во-первых, после применения формулы всё выражение нужно заключить в большие скобки , так как перед исходным интегралом находится константа . Не теряем её ! Скобки можно раскрыть на любом дальнейшем шаге, я это сделал в самую последнюю очередь. В первом «куске» проявляем крайнюю аккуратность в подстановке, как видите, константа не при делах, и пределы интегрирования подставляются в произведение . Данное действие выделено квадратными скобками. Ну а интеграл второго «куска» формулы вам хорошо знаком из тренировочного задания;-)

И самое главное – предельная концентрация внимания!

3) Ищем третий коэффициент Фурье:

Получен родственник предыдущего интеграла, который тоже интегрируется по частям :

Этот экземпляр чуть сложнее, закомментирую дальнейшие действия пошагово:

(1) Выражение полностью заключаем в большие скобки . Не хотел показаться занудой, слишком уж часто теряют константу .

(2) В данном случае я немедленно раскрыл эти большие скобки. Особое внимание уделяем первому «куску»: константа курит в сторонке и не участвует в подстановке пределов интегрирования ( и ) в произведение . Ввиду загромождённости записи это действие снова целесообразно выделить квадратными скобками. Со вторым «куском» всё проще: здесь дробь появилась после раскрытия больших скобок, а константа – в результате интегрирования знакомого интеграла;-)

(3) В квадратных скобках проводим преобразования , а в правом интеграле – подстановку пределов интегрирования.

(4) Выносим «мигалку» из квадратных скобок: , после чего раскрываем внутренние скобки: .

(5) Сокращаем 1 и –1 в скобках, проводим окончательные упрощения.

Наконец-то найдены все три коэффициента Фурье:

Подставим их в формулу :

При этом не забываем разделить пополам. На последнем шаге константа («минус два»), не зависящая от «эн», вынесена за пределы суммы.

Таким образом, мы получили разложение функции в ряд Фурье на промежутке :

Изучим вопрос сходимости ряда Фурье. Я объясню теорию, в частности теорему Дирихле , буквально «на пальцах», поэтому если вам необходимы строгие формулировки, пожалуйста, обратитесь к учебнику по математическому анализу (например, 2-й том Бохана; или 3-й том Фихтенгольца, но в нём труднее) .

Во второй части задачи требуется изобразить график , график суммы ряда и график частичной суммы .

График функции представляет собой обычную прямую на плоскости , которая проведена чёрным пунктиром:

Разбираемся с суммой ряда . Как вы знаете, функциональные ряды сходятся к функциям. В нашем случае построенный ряд Фурье при любом значении «икс» сойдётся к функции , которая изображена красным цветом. Данная функция терпит разрывы 1-го рода в точках , но определена и в них (красные точки на чертеже)

Таким образом: . Легко видеть, что заметно отличается от исходной функции , именно поэтому в записи ставится значок «тильда», а не знак равенства.

Изучим алгоритм, по которому удобно строить сумму ряда.

На центральном интервале ряд Фурье сходится к самой функции (центральный красный отрезок совпадает с чёрным пунктиром линейной функции).

Теперь немного порассуждаем о природе рассматриваемого тригонометрического разложения. В ряд Фурье входят только периодические функции (константа, синусы и косинусы), поэтому сумма ряда тоже представляет собой периодическую функцию .

Что это значит в нашем конкретном примере? А это обозначает то, что сумма ряда – непременно периодична и красный отрезок интервала обязан бесконечно повторяться слева и справа.

Думаю, сейчас окончательно прояснился смысл фразы «период разложения ». Упрощённо говоря, через каждые ситуация вновь и вновь повторяется.

На практике обычно достаточно изобразить три периода разложения, как это сделано на чертеже. Ну и ещё «обрубки» соседних периодов – чтобы было понятно, что график продолжается.

Особый интерес представляют точки разрыва 1-го рода . В таких точках ряд Фурье сходится к изолированным значениям, которые расположены ровнёхонько посередине «скачка» разрыва (красные точки на чертеже). Как узнать ординату этих точек? Сначала найдём ординату «верхнего этажа»: для этого вычислим значение функции в крайней правой точке центрального периода разложения: . Чтобы вычислить ординату «нижнего этажа» проще всего взять крайнее левое значение этого же периода: . Ордината среднего значения – это среднее арифметическое суммы «верха и низа»: . Приятным является тот факт, что при построении чертежа вы сразу увидите, правильно или неправильно вычислена середина.

Построим частичную сумму ряда и заодно повторим смысл термина «сходимость». Мотив известен ещё из урока о сумме числового ряда . Распишем наше богатство подробно:

Чтобы составить частичную сумму необходимо записать нулевой + ещё два члена ряда. То есть,

На чертеже график функции изображен зелёным цветом, и, как видите, он достаточно плотно «обвивает» полную сумму . Если рассмотреть частичную сумму из пяти членов ряда , то график этой функции будет ещё точнее приближать красные линии, если сто членов – то «зелёный змий» фактически полностью сольётся с красными отрезками и т.д. Таким образом, ряд Фурье сходится к своей сумме .

Интересно отметить, что любая частичная сумма – это непрерывная функция , однако полная сумма ряда всё же разрывна.

На практике не так уж редко требуется построить и график частичной суммы. Как это сделать? В нашем случае необходимо рассмотреть функцию на отрезке , вычислить её значения на концах отрезка и в промежуточных точках (чем больше точек рассмотрите – тем точнее будет график). Затем следует отметить данные точки на чертеже и аккуратно изобразить график на периоде , после чего «растиражировать» его на соседние промежутки. А как иначе? Ведь приближение – это тоже периодическая функция… …чем-то мне её график напоминает ровный ритм сердца на дисплее медицинского прибора.

Выполнять построение, конечно, не сильно удобно, так как и приходится проявлять сверхаккуратность, выдерживая точность не меньше, чем до половины миллиметра. Впрочем, читателей, которые не в ладах с черчением, обрадую – в «реальной» задаче выполнять чертёж нужно далеко не всегда, где-то в 50% случаев требуется разложить функцию в ряд Фурье и всё.

После выполнения чертежа завершаем задание:

Ответ :

Во многих задачах функция терпит разрыв 1-го рода прямо на периоде разложения:

Пример 3

Разложить в ряд Фурье функцию , заданную на отрезке . Начертить график функции и полной суммы ряда.

Предложенная функция задана кусочным образом (причём, заметьте, только на отрезке ) и терпит разрыв 1-го рода в точке . Можно ли вычислить коэффициенты Фурье? Без проблем. И левая и правая части функции интегрируемы на своих промежутках, поэтому интегралы в каждой из трёх формул следует представить в виде суммы двух интегралов. Посмотрим, например, как это делается у нулевого коэффициента:

Второй интеграл оказался равным нулю, что убавило работы, но так бывает далеко не всегда.

Аналогично расписываются два других коэффициента Фурье.

Как изобразить сумму ряда? На левом интервале чертим отрезок прямой , а на интервале – отрезок прямой (жирно-жирно выделяем участок оси ). То есть, на промежутке разложения сумма ряда совпадает с функцией везде, кроме трёх «нехороших» точек. В точке разрыва функции ряд Фурье сойдётся к изолированному значению, которое располагается ровно посередине «скачка» разрыва. Его нетрудно увидеть и устно: левосторонний предел: , правосторонний предел: и, очевидно, что ордината средней точки равна 0,5.

В силу периодичности суммы , картинку необходимо «размножить» на соседние периоды, в частности изобразить то же самое на интервалах и . При этом, в точках ряд Фурье сойдётся к срединным значениям.

По сути-то ничего нового здесь нет.

Постарайтесь самостоятельно справиться с данной задачей. Примерный образец чистового оформления и чертёж в конце урока.

Разложение функции в ряд Фурье на произвольном периоде

Для произвольного периода разложения , где «эль» – любое положительное число, формулы ряда Фурье и коэффициентов Фурье отличаются немного усложнённым аргументом синуса и косинуса:

Если , то получаются формулы промежутка , с которых мы начинали.

Алгоритм и принципы решения задачи полностью сохраняются, но возрастает техническая сложность вычислений:

Пример 4

Разложить функцию в ряд Фурье и построить график суммы.

Решение : фактически аналог Примера №3 с разрывом 1-го рода в точке . В данной задаче период разложения , полупериод . Функция определена только на полуинтервале , но это не меняет дела – важно, что оба куска функции интегрируемы.

Разложим функцию в ряд Фурье:

Поскольку функция разрывна в начале координат, то каждый коэффициент Фурье очевидным образом следует записать в виде суммы двух интегралов:

1) Первый интеграл распишу максимально подробно:

2) Тщательным образом вглядываемся в поверхность Луны:

Второй интеграл берём по частям :

На что следует обратить пристальное внимание, после того, как мы звёздочкой открываем продолжение решения?

Во-первых, не теряем первый интеграл , где сразу же выполняем подведение под знак дифференциала . Во-вторых, не забываем злополучную константу перед большими скобками и не путаемся в знаках при использовании формулы . Большие скобки, всё-таки удобнее раскрывать сразу же на следующем шаге.

Остальное дело техники, затруднения может вызвать только недостаточный опыт решенияинтегралов.

Да, не зря именитые коллеги французского математика Фурье возмущались – как это тот посмел раскладывать функции в тригонометрические ряды?! =) К слову, наверное, всем интересен практический смысл рассматриваемого задания. Сам Фурье работал над математической моделью теплопроводности, а впоследствии ряд, названный его именем стал применяться для исследования многих периодических процессов, коих в окружающем мире видимо-невидимо. Сейчас, кстати, поймал себя на мысли, что не случайно сравнил график второго примера с периодическим ритмом сердца. Желающие могут ознакомиться с практическим применением преобразования Фурье в сторонних источниках. …Хотя лучше не надо – будет вспоминаться, как Первая Любовь =)

3) Учитывая неоднократно упоминавшиеся слабые звенья, разбираемся с третьим коэффициентом:

Интегрируем по частям:

Подставим найдённые коэффициенты Фурье в формулу , не забывая поделить нулевой коэффициент пополам:

Построим график суммы ряда. Кратко повторим порядок действий: на интервале строим прямую , а на интервале – прямую . При нулевом значении «икс» ставим точку посередине «скачка» разрыва и «тиражируем» график на соседние периоды:


На «стыках» периодов сумма также будет равна серединам «скачка» разрыва .

Готово. Напоминаю, что сама функция по условию определена только на полуинтервале и, очевидно, совпадает с суммой ряда на интервалах

Ответ :

Иногда кусочно-заданная функция бывает и непрерывна на периоде разложения. Простейший образец: . Решение (см. 2-й том Бохана) такое же, как и двух предыдущих примерах: несмотря на непрерывность функции в точке , каждый коэффициент Фурье выражается суммой двух интегралов.

На промежутке разложения точек разрыва 1-го рода и/или точек «стыка» графика может быть и больше (две, три и вообще любое конечное количество). Если функция интегрируема на каждой части, то она также разложима в ряд Фурье. Но из практического опыта такую жесть что-то не припоминаю. Тем не менее, встречаются более трудные задания, чем только что рассмотренное, и в конце статьи для всех желающих есть ссылки на ряды Фурье повышенной сложности.

А пока расслабимся, откинувшись в креслах и созерцая бескрайние звёздные просторы:

Пример 5

Разложить функцию в ряд Фурье на промежутке и построить график суммы ряда.

В данной задаче функция непрерывна на полуинтервале разложения, что упрощает решение. Всё очень похоже на Пример №2. С космического корабля никуда не деться – придётся решать =) Примерный образец оформления в конце урока, график прилагается.

Разложение в ряд Фурье чётных и нечётных функций

С чётными и нечётными функциями процесс решения задачи заметно упрощается. И вот почему. Вернёмся к разложению функции в ряд Фурье на периоде «два пи» и произвольном периоде «два эль» .

Предположим, что наша функция чётна. Общий же член ряда, как вы видите, содержит чётные косинусы и нечётные синусы. А если мы раскладываем ЧЁТНУЮ функцию, то зачем нам нечётные синусы?! Давайте обнулим ненужный коэффициент: .

Таким образом, чётная функция раскладывается в ряд Фурье только по косинусам :

Поскольку интегралы от чётных функций по симметричному относительно нуля отрезку интегрирования можно удваивать, то упрощаются и остальные коэффициенты Фурье.

Для промежутка :

Для произвольного промежутка:

К хрестоматийным примерам, которые есть практически в любом учебнике по матанализу, относятся разложения чётных функций . Кроме того, они неоднократно встречались и в моей личной практике:

Пример 6

Дана функция . Требуется:

1) разложить функцию в ряд Фурье с периодом , где – произвольное положительное число;

2) записать разложение на промежутке , построить функцию и график полной суммы ряда .

Решение : в первом пункте предлагается решить задачу в общем виде, и это очень удобно! Появится надобность – просто подставьте своё значение.

1) В данной задаче период разложения , полупериод . В ходе дальнейших действий, в частности при интегрировании, «эль» считается константой

Функция является чётной, а значит, раскладывается в ряд Фурье только по косинусам: .

Коэффициенты Фурье ищем по формулам . Обратите внимание на их безусловные преимущества. Во-первых, интегрирование проводится по положительному отрезку разложения, а значит, мы благополучно избавляемся от модуля , рассматривая из двух кусков только «икс». И, во-вторых, заметно упрощается интегрирование.

Два:

Интегрируем по частям:

Таким образом:
, при этом константу , которая не зависит от «эн», выносим за пределы суммы.

Ответ :

2) Запишем разложение на промежутке , для этого в общую формулу подставляем нужное значение полупериода :

Ряд Фурье четной периодической функции f (x) с периодом 2р содержит только члены с косинусами (т.е. не содержит членов с синусами) и может включать постоянный член. Следовательно,

где коэффициенты ряда Фурье,

Ряд Фурье нечетной периодической функции f (x) с периодом 2р содержит только члены с синусами (т.е. не содержит членов с косинусами).

Следовательно,

где коэффициенты ряда Фурье,

Если функция определена для диапазона, скажем от 0 до р, а не только от 0 до 2р, ее можно разложить в ряд только по синусам или тольо по косинусам. Полученный ряд Фурье называется рядом Фурье на полупериоде.

Если требуется получить разложение Фурье на полупериоде по косинусам функции f (x) в диапазоне от 0 до р, то необходимо составить четную периодическую функцию. На рис. ниже показана функция f (x) =х, построенная на интервале от х=0 до х=р. Поскольку четная функция симметрична относительно оси f (x), проводим линию АВ, как показано на рис. ниже. Если предположить, что за пределами рассмотренного интервала полученная треугольная форма является периодической с периодом 2р, то итоговый график имеет вид, показ. на рис. ниже. Поскольку требуется получить разложение Фурье по косинусам, как и ранее, вычисляем коэффициенты Фурье a o и a n


Если требуется получить разложение Фурье на полупериоде по синусам функции f (x) в диапазоне от 0 до р, то необходимо составить нечетную периодическую функцию. На рис. ниже показана функция f (x) =x, построенная на интервале от от х=0 до х=р. Поскольку нечетная функция симметрична относительно начала координат, строим линию CD, как показано на рис.

Если предположить, что за пределами рассмотренного интервала полученный пилообразный сигнал является периодическим с периодом 2р, то итоговый график имеет вид, показанный на рис. Поскольку требуется получить разложение Фурие на полупериоде по синусам, как и ранее, вычисляем коэффициент Фурье. b

Функции, разлагая их на компоненты. Переменные токи и напряжения, смещения, скорость и ускорение кривошипно-шатунных механизмов и акустические волны — это типичные практические примеры применения периодических функций в инженерных расчетах.

Разложение в ряд Фурье основывается на предположении, что все имеющие практическое значение функции в интервале -π ≤x≤ π можно выразить в виде сходящихся тригонометрических рядов (ряд считается сходящимся, если сходится последовательность частичных сумм, составленных из его членов):

Стандартная (=обычная) запись через сумму sinx и cosx

f(x)=a o + a 1 cosx+a 2 cos2x+a 3 cos3x+. ..+b 1 sinx+b 2 sin2x+b 3 sin3x+…,

где a o , a 1 ,a 2 ,…,b 1 ,b 2 ,.. — действительные константы, т.е.

Где для диапазона от -π до π коэффициенты ряда Фурье рассчитываются по формулам:

Коэффициенты a o ,a n и b n называются коэффициентами Фурье , и если их можно найти, то ряд (1) называется рядом Фурье, соответствующим функции f(x). Для ряда (1) член (a 1 cosx+b 1 sinx) называется первой или основной гармоникой,

Другой способ записи ряда — использование соотношения acosx+bsinx=csin(x+α)

f(x)=a o +c 1 sin(x+α 1)+c 2 sin(2x+α 2)+…+c n sin(nx+α n)

Где a o — константа, с 1 =(a 1 2 +b 1 2) 1/2 , с n =(a n 2 +b n 2) 1/2 — амплитуды различных компонент, а равен a n =arctg a n /b n .

Для ряда (1) член (a 1 cosx+b 1 sinx) или c 1 sin(x+α 1) называется первой или основной гармоникой, (a 2 cos2x+b 2 sin2x) или c 2 sin(2x+α 2) называется второй гармоникой и так далее.

Для точного представления сложного сигнала обычно требуется бесконечное количество членов. Однако во многих практических задачах достаточно рассмотреть только несколько первых членов.

Ряд Фурье непериодических функций с периодом 2π.
Разложение непериодических функций в ряд Фурье.

Если функция f(x) непериодическая, значит, она не может быть разложена в ряд Фурье для всех значений х. Однако можно определить ряд Фурье, представляющий функцию в любом диапазоне шириной 2π.

Если задана непериодическая функция, можно составить новую функцию, выбирая значения f(x) в определенном диапазоне и повторяя их вне этого диапазона с интервалом 2π. Поскольку новая функция является периодической с периодом 2π, ее можно разложить в ряд Фурье для всех значений х. Например, функция f(x)=x не является периодической. Однако, если необходимо разложить ее в ряд Фурье на интервале от о до 2π, тогда вне этого интервала строится периодическая функция с периодом 2π (как показано на рис. ниже) .

Для непериодических функций, таких как f(x)=х, сумма ряда Фурье равна значению f(x) во всех точках заданного диапазона, но она не равна f(x) для точек вне диапазона. Для нахождения ряда Фурье непериодической функции в диапазоне 2π используется все таже формула коэффициентов Фурье.

Четные и нечетные функции.

Говорят, функция y=f(x) четная , если f(-x)=f(x) для всех значений х. Графики четных функций всегда симметричны относительно оси у (т.е. являются зеркально отраженными). Два примера четных функций: у=х 2 и у=cosx.

Говорят, что функция y=f(x) нечетная, если f(-x)=-f(x) для всех значений х. Графики нечетных функций всегда симметричны относительно начала координат.

Многие функции не являются ни четными, ни нечетными.

Разложение в ряд Фурье по косинусам.

Ряд Фурье четной периодической функции f(x) с периодом 2π содержит только члены с косинусами (т.е. не содержит членов с синусами) и может включать постоянный член. Следовательно,

где коэффициенты ряда Фурье,

Ряд Фурье нечетной периодической функции f(x) с периодом 2π содержит только члены с синусами (т. е. не содержит членов с косинусами).

Следовательно,

где коэффициенты ряда Фурье,

Ряд Фурье на полупериоде.

Если функция определена для диапазона, скажем от 0 до π, а не только от 0 до 2π, ее можно разложить в ряд только по синусам или тольо по косинусам. Полученный ряд Фурье называется рядом Фурье на полупериоде.

Если требуется получить разложение Фурье на полупериоде по косинусам функции f(x) в диапазоне от 0 до π, то необходимо составить четную периодическую функцию. На рис. ниже показана функция f(x)=х, построенная на интервале от х=0 до х=π. Поскольку четная функция симметрична относительно оси f(x), проводим линию АВ, как показано на рис. ниже. Если предположить, что за пределами рассмотренного интервала полученная треугольная форма является периодической с периодом 2π, то итоговый график имеет вид, показ. на рис. ниже. Поскольку требуется получить разложение Фурье по косинусам, как и ранее, вычисляем коэффициенты Фурье a o и a n

Если требуется получить функции f(x) в диапазоне от 0 до π, то необходимо составить нечетную периодическую функцию. На рис. ниже показана функция f(x)=x, построенная на интервале от от х=0 до х=π. Поскольку нечетная функция симметрична относительно начала координат, строим линию CD, как показано на рис. Если предположить, что за пределами рассмотренного интервала полученный пилообразный сигнал является периодическим с периодом 2π, то итоговый график имеет вид, показанный на рис. Поскольку требуется получить разложение Фурие на полупериоде по синусам, как и ранее, вычисляем коэффициент Фурье. b

Ряд Фурье для произвольного интервала.

Разложение периодической функции с периодом L.

Периодическая функция f(x) повторяется при увеличении х на L, т.е. f(x+L)=f(x). Переход от рассмотренных ранее функций с периодом 2π к функциям с периодом L довольно прост, поскольку его можно осуществить с помощью замены переменной.

Чтобы найти ряд Фурье функции f(x) в диапазоне -L/2≤x≤L/2, введем новую переменную u таким образом, чтобы функция f(x) имела период 2π относительно u. Если u=2πх/L, то х=-L/2 при u=-π и х=L/2 при u=π. Также пусть f(x)=f(Lu/2π)=F(u). Ряд Фурье F(u) имеет вид

Где коэффициенты ряда Фурье,

Однако чаще приведенную выше формулу приводят к зависимости от х. Поскольку u=2πх/L, значит, du=(2π/L)dx, а пределы интегрирования — от -L/2 до L/2 вместо — π до π. Следовательно, ряд Фурье для зависимости от х имеет вид

где в диапазоне от -L/2 до L/2 коэффициенты ряда Фурье,

(Пределы интегрирования могут быть заменены на любой интервал длиной L, например, от 0 до L)

Ряд Фурье на полупериоде для функций, заданных в интервале L≠2π.

Для подстановки u=πх/L интервал от х=0 до х=L соответствует интервалу от u=0 до u=π. Следовательно, функцию можно разложить в ряд только по косинусам или только по синусам, т.е. в ряд Фурье на полупериоде .

Разложение по косинусам в диапазоне от 0 до L имеет вид

4.5. Разложение в ряд Фурье только по синусам или только по косинусам

Начнем с простого замечания: если заданная на отрезке интегрируемая функция является нечетной, то есть для всех выполняется равенство , то .

Для четной функции справедливо .

Напомним некоторые свойства четных и нечетных функций на :

    1. Произведение двух четных или двух нечетных функций есть функция четная;

    2. Произведение четной и нечетной функций есть нечетная функция.

Утверждение. Пусть определена и интегрируема на , а -ее коэффициенты Фурье. Тогда

      1. если -нечетная, то

, а ряд Фурье имеет вид .

      1. если — четная, то

, а ряд Фурье имеет вид .

Допустим, что функция задана на отрезке . Если мы хотим найти разложение на этом отрезке в ряд Фурье, то сначала продолжим на симметричный промежуток произвольным образом, а потом воспользуемся формулами для коэффициентов Фурье.

Если продолжить функцию четным образом, то получим разложение только по косинусам, а если продолжить нечетным образом, то – только по синусам. При этом в первом случае продолженная функция будет иметь вид

,

а во втором случае

4.6. Разложение в ряд Фурье функции, заданной на произвольном промежутке

Пусть задана на отрезке , и на этом отрезке она кусочно-гладкая. Рассмотрим периодическую кусочно-гладкую функцию с периодом

,

которая совпадает с на , а -произвольная кусочно-гладкая функция.

Таким образом, была продолжена на симметричный отрезок. Теперь для существует разложение в ряд Фурье. Сумма этого ряда совпадает с во всех точках непрерывности отрезка , то есть функция разложена в ряд Фурье на .

Алгоритм разложения функции в тригонометрический ряд Фурье:

  1. выяснить формально ряд Фурье по заданию функции;

  2. найти коэффициенты ряда Фурье;

  3. используя теорему о достаточном условии сходимости ряда Фурье, найти сумму ряда, построить график и . Выяснить, в каких точках совпадает с .

4.7. Контрольные вопросы и задания.

  1. Какая функция называется периодической? Является ли функция Дирихле периодической? Чему равен период? Имеет ли эта функция основной период?

  1. Что такое тригонометрический ряд?

  2. Какой тригонометрический ряд называется рядом Фурье?

  1. Являются ли тригонометрические ряды и рядами Фурье?

  1. Сформулировать достаточные условия поточечной сходимости ряда Фурье.

  2. Записать равенство Парсеваля и неравенство Бесселя для тригонометрического ряда Фурье.

  3. Какой вид имеет ряд Фурье для нечетной интегрируемой функции?

  4. Какой вид имеет ряд Фурье для -периодической функции?

4.

8. Образцы решения типовых задач

При нахождении коэффициентов Фурье полезно помнить:

.

Пример 1. Разложить функцию в ряд Фурье на интервале . Построить график суммы ряда Фурье. Вычислить суммы получающихся рядов, полагая .

Построим график данной функции:

Продолжим данную функцию периодически с периодом на всю прямую.

Построим график суммы ряда Фурье

Найдём коэффициенты ряда Фурье. Так как нечётная на

Итак, .

Используя полученное разложение с учётом вида графика суммы ряда Фурье, из которого видно, к чему сходится ряд в точках разрыва, найдём суммы некоторых числовых рядов.

При получим .

При получим

.

При получим

.

Пример 2. Разложить в ряд Фурье по косинусам функцию (полупериод функции равен )

Изобразим график заданной функции

Продолжим функцию чётным образом на промежутке , тогда коэффициенты .

Продолжим полученную функцию с периодом на всю прямую. Так как продолжение будет непрерывной функцией, то для график суммы ряда Фурье совпадает с графиком продолженной функции

Вычислим коэффициенты ряда Фурье

при .

Пример 3. Разложить в ряд Фурье по синусам функцию (полупериод функции равен )

Разложение функции в ряд по синусам — это ряд Фурье нечётного продолжения функции с промежутка на промежуток .

Изобразим график суммы ряда Фурье

Имеем .

Формула ряда Фурье — Что такое формула ряда Фурье?

Формула ряда Фурье дает разложение периодической функции f(x) через бесконечную сумму синусов и косинусов. Он используется для разложения любой периодической функции или периодического сигнала на сумму набора простых осциллирующих функций, а именно синусов и косинусов. Давайте разберемся с формулой ряда Фурье на решенных примерах.

Что такое формулы ряда Фурье?

В рядах Фурье используются ортогональные соотношения функций косинуса и синуса. Формула ряда Фурье для функции имеет вид 9{\pi}f(x)sin\;nx\;dx\)

  • n = 1, 2, 3…..
  • Есть вопросы по основным математическим понятиям?

    Станьте чемпионом по решению проблем, используя логику, а не правила. Узнайте, почему математика стоит за нашими сертифицированными экспертами.

    Примеры формул ряда Фурье

    9{2}}\)

    Часто задаваемые вопросы о формулах ряда Фурье

    Что подразумевается под рядом Фурье?

    Ряд Фурье представляет собой разложение периодической функции f(x) по бесконечной сумме синусов и косинусов. {∞} B_n sin(nπx/L)

    Каково применение формулы ряда Фурье?

    Ряд Фурье описывает периодический сигнал в терминах косинуса и синусоиды. Таким образом, он моделирует любой произвольный периодический сигнал комбинацией синусов и косинусов.

    Как решить ряд Фурье, используя формулу ряда Фурье?

    Шаги для решения ряда Фурье приведены ниже:
    Шаг 1: Умножьте заданную функцию на синус или косинус, затем проинтегрируйте
    . Шаг 2: Оценка для n=0, n=1 и т. д., чтобы получить значения коэффициентов.
    Шаг 3: Наконец, подставляем все коэффициенты в формулу Фурье.

    Какие два типа формул ряда Фурье существуют?

    Существует два типа формул ряда Фурье: формула тригонометрического ряда и формула экспоненциального ряда.

     

    Определение ряда Фурье и типичные примеры

    Барон Жан Батист Жозеф Фурье (1768–1830) выдвинул идею о том, что любая периодическая функция может быть представлена ​​последовательностью синусов и косинусов, которые находятся в гармонической взаимосвязи.

    Рис.1 Барон Жан Батист Жозеф Фурье (1768−1830)

    Для более подробного рассмотрения этой идеи необходимо ввести некоторые определения и общие термины.

    Основные определения

    Говорят, что функция \(f\left( x \right)\) имеет период \(P\), если \(f\left( {x + P} \right) = f\left( x \right)\ ) для всех \(x.\) Пусть функция \(f\left( x \right)\) имеет период \(2\pi.\) В этом случае достаточно рассмотреть поведение функции на отрезке \(\left[ { — \pi ,\pi } \right].\) 9\ пи {\ влево | {f\left( x \right)} \right|dx} \lt \infty ;\]

  • Предположим также, что функция \(f\left( x \right)\) является однозначной, кусочно-непрерывной (должна иметь конечное число скачков) и кусочно-монотонной (должна иметь конечное число максимумов и минимумов) .
  • При выполнении условий \(1\) и \(2\) ряд Фурье для функции \(f\left( x \right)\) существует и сходится к заданной функции (см. также Сходимость Страница ряда Фурье об условиях сходимости.) 9\pi {f\left( x \right)\sin nxdx} . \pi {1dx} = \frac{1}{\pi } \cdot \pi = 1.\] 95}}}{{5\pi }}\sin 5x + \ldots = \frac{1}{2} + \frac{2}{\pi}\sin x + \frac{2}{{3\pi }}\sin 3x + \frac{2}{{5\pi }}\sin 5x + \ldots\]

    График функции и разложения в ряд Фурье для \(n = 10\) показаны ниже на рисунке \(2.\)

    Рисунок 2, n = 10

    См. больше задач на стр. 2.

    Ряд функций Фурье с произвольным периодом

    Разложение в ряд Фурье на интервале [−

    L , L ]

    Предположим, что функция f ( x ) кусочно-непрерывна на интервале [− L , L ]. Используя замену x = Ly / π (− π x π ), мы можем преобразовать его в функцию

    \[F\влево(y\вправо) = f\влево({\frac{{Ly}}{\pi}} \вправо),\]

    , который определен и интегрируем на [− π , π ]. Разложение этой функции в ряд Фурье 9L {f\left( x \right)\sin\frac{{n\pi x}}{L}dx} .\]

    Решенные проблемы

    Щелкните или коснитесь проблемы, чтобы увидеть решение.

    Пример 1

    Найдите ряд Фурье функции

    \[ {е\влево(х\вправо)}= {\ начать {случаи} А, & 0 \ле х \ле L \\ 0, & L \lt x \le 2L \end{случаи}.} \]

    Пример 2

    Найдите ряд Фурье функции:

    \[ {е\влево(х\вправо)}= {\ начать {случаи} 0,&-1 \ле х \ле 0 \\ х, & 0 \lt х \ле 1 \end{случаи}.} \] 9{n + 1}}}}{{n\pi }}\sin n\pi x} \right]} .\]

    Рисунок 2, n = 5, n = 10

    См. больше задач на стр. 2.

    Формула ряда Фурье — GeeksforGeeks

    Ряд Фурье представляет собой сумму синусоидальных и косинусоидальных волн, которая представляет собой периодическую функцию. Каждая волна в сумме, или гармоника, имеет частоту, кратную основной частоте периодической функции. Гармонический анализ может использоваться для определения фазы и амплитуды каждой гармоники. Ряд Фурье может иметь неограниченное количество гармоник. Суммирование некоторых, но не всех гармоник в ряду Фурье функции дает приближение к этой функции. Например, прямоугольную волну можно аппроксимировать, используя первые несколько гармоник ряда Фурье.

    Ряд Фурье

    Как было показано выше, периодические функции часто появляются в задачах по высшей математике. Способ решения этих проблем состоит в том, чтобы представить их в терминах основных периодических функций, которые имеют небольшой диапазон и могут иметь область определения всех действительных чисел, таких как синус и косинус; это приводит нас к ряду Фурье (FS). Ряд Фурье — особенно полезный инструмент для работы с уравнениями в частных производных.

    Предположим, что нам дана периодическая функция f(x). Теперь, поскольку исходная функция является периодической, следовательно,

    c 1 ​f 1 ​(x) + … + c n ​f n ​(x)​

    Далее рассмотрим бесконечный ряд,

    ⇒ 900 (3) ⇒ 900 (1) 2L-периодическая функция сходится при всех x, то функция, к которой она сходится, будет периодической периода 2L. Теперь, как показано выше, нам нужно представить функцию f(x) таким образом, чтобы периодическая функция f(x) была заменена такими функциями, как синус и косинус. Для этого ряд Фурье задается выражением

     

    Здесь

     

    .

     .

    n = 1,2,3….
       

    Общая форма ряда Фурье

    Для любой функции f(x) с периодом 2L ряд Фурье задается как

     .

    Экспоненциальная форма ряда Фурье

    Из приведенного выше уравнения

    .

    Теперь по формуле Эйлера

    e = cosθ +isinθ

    Используя это

    f(x) = C n e inx .

    Здесь Cn называется коэффициентом разложения и рассчитывается как

     .

    Условия для рядов Фурье

    Предположим, что функция f(x) имеет период 2π и интегрируема в периоде [-π, π]. Теперь есть два условия.

    Функция f(x) с периодом 2π абсолютно интегрируема на [-π, π], так что следующий интеграл Дирихле от этой функции конечен:

    Следующее условие состоит в том, что функция является однозначной, кусочно-непрерывной (должна иметь конечное число скачков) и кусочно-монотонной (должна иметь конечное число максимумов и минимумов).

    При выполнении условий 1 и 2 ряд Фурье для функции существует и сходится к заданной функции. Это означает, что сумма ряда Фурье любой заданной функции сходится обратно, чтобы дать ту же самую функцию. Это основное определение Расширение ряда Фурье. Перед дальнейшим пониманием концепции ряда Фурье мы должны сначала понять концепцию нечетных и четных функций и периодических функций.

    • Нечетная функция: Предположим, нам дана функция y = f(x).

    Теперь, если

    f(-x) = -f(x) = -y

    , то функция называется нечетной.

    Функция имеет нечетный характер и симметрична относительно начала координат

    • Четная функция : Снова рассмотрим функцию f(x) = y.

    Если f(-x) = f(x) = y

    Тогда функция четна по своей природе.

    Это четная функция, график которой симметричен относительно оси Y.

    • Периодические функции: Пусть функция f(x) периодична с интервалом λ. Теперь рассмотрим элемент x как часть области определения этой функции. Это означает, что

    f(x) = f(x + λ).

    График функции tanx является примером периодической функции.

    Следовательно, периодические функции — это те функции, которые повторяются в интервале значений (λ, как показано выше). Наименьшее возможное положительное значение λ называется периодом этой функции.

    Ряд Фурье

    Примеры задач

    Вопрос 1: Найдите разложение в ряд Фурье функции f(x) = e x в пределах  [– π, π].

    Решение :

    Использование разложения в ряд Фурье.

     .

     .

     .

    Ряд Фурье для этой функции задается как 

     .

    Вопрос 2: Найдите разложение в ряд Фурье функции f(x) = x в пределах  [– 1, 1].

    Решение:

    Из разложения в ряд Фурье. Здесь

    .

     .

     .

     .

     .

    Ом решая интегралы, мы получаем четные функции и одну нечетную функцию. Следовательно,

    .

    Вопрос 3. Предположим, что функция f(x) = tanx находит свое разложение Фурье в пределах [-π, π].

    Решение:

     

     

       

    Теперь невозможно найти интеграл от tanx⋅sinnx и tanx⋅.

    Следовательно, ряд Фурье для этой функции f(x) = tanx не определен.

    Вопрос 4. Найдите ряд Фурье функции f(x) = 1 для пределов  [– π, π] .

    Решение:

    по сравнению с общим расширением серии Фурье. 5: Рассмотрим функцию f(x) = x 2 для пределов  [– π, π]. Найдите его разложение в ряд Фурье.

    Решение:

    Сравнивая с общим разложением в ряд Фурье, получаем,

    Вопрос 6: Найдите разложение в ряд Фурье функции f(x) = 4-3x для пределов  [– 1, 1].

    Решение:

    Сравнивая с общим разложением в ряд Фурье, получаем

    Вопрос 7: Найдите разложение функции . Для пределов  [– π, π].

    Решение:

    Сравнивая с общим расширением ряда Фурье, мы получаем,


    Расширение ряда (2): ряд Фурье

    Расширение ряда (2): ряд Фурье

    Расширение серии 5.10 (2): серия Фурье

    5.10.1 Ряды Тейлора и ряды Фурье

    Частичные суммы ряда Тейлора, приближающие функцию f(x) в окрестности расчетная точка x0 через частичные суммы степенного ряда. Если кто-то хочет аппроксимировать функцию над Для большего интервала потребуются члены очень высокого порядка. Полином, полученный усечение ряда Тейлора должно иметь как минимум столько поворотных точек, сколько функция. Для периодических функций это было бы очень утомительно для больших интервалов. чем период.

    Периодические функции имеют большое практическое значение в телекоммуникациях и электротехника. Для таких функций аппроксимация через суперпозиция периодических стандартных функций (синуса и косинуса0 гораздо лучше подходит. On расширяет функцию в ряд, который состоит из основного тона и обертонов, т. е. из функций грех и cosnx с целым числом значения н.

    Сразу очевидна аналогия с анализом колеблющейся струны: sinx описывает вибрацию основного тона, sin2x что у октавы, sin3x что из квинты выше октавы и так далее. Для струны, закрепленной на обоих концах; переменная х теперь произведение ωt угловая частота ω и время т.

    x=ωt=2πνt=2πtT;νчастота колебаний;T продолжительность одного периода

    Фурье

    В зависимости от формы f(t) накладывают большее или меньшее количество этих синусоидальных/косинусоидальных колебаний с определенной силы и выражается в виде числа, определяющего амплитуду. Набор амплитуды обертонов, т.е. коэффициенты разложения ряда представляют собой спектр периодических колебаний. Спектр и форма колебаний соответствующие представления об одном и том же явлении. Это представление в терминах функций синусов и косинусов называется рядом Фурье ф(т).

    В то время как частичные суммы ряда Тейлора аппроксимируют функцию в близости точки , частичные суммы ряда Фурье являются приближениями для весь интервал основного периода и, следовательно, также — из-за периодичность рассматриваемых функций — для неограниченной области переменной Икс. Ряд Фурье не обязательно должен совпадать с функцией ни в одной точке, а это случай для ряда Тейлора в точке вычисления.

    Зависит от свойств f(t), сколько обертонов нужно наложить, чтобы аппроксимировать функцию почти все точки. Если трактовать понятие сходимости не строго, то ряды Фурье сходятся для всех функций, даже для непрерывных. Конвергенция тогда не обязательно монотонно, т.е. может быть лучше для некоторых значений т и хуже для некоторых другие значения t и даже потерпеть неудачу для некоторых значений! На разрывах наблюдаются перерегулирования даже для высших порядков ряда. В телекоммуникациях это называется звонком.

    Поскольку периодические явления, которые мы здесь рассматриваем, в основном представляют собой колебания во времени, переменной обычно является x=ωt. Чтобы также смоделировать фазы отдельных обертонов, мы используем сумму членов с грех и cosnx. затем сумма представляет собой сдвинутую по фазе функцию синуса или косинуса. Таким образом, общий Фурье серия читает

    f(t)=a02+ ∑n=1∞an cos(nωt)+bn sin(nωt).

    Для заданного спектра a0,ai,bi,i=1,2,⋯ можно вычислить f(t). Для заданной функции f(t) все коэффициенты могут быть определены и, таким образом, известен спектр.

    5.10.2 Определение коэффициентов Фурье

    Как теперь получить коэффициенты а бн?

    Для ряда Тейлора мы использовали тот факт, что после дифференцирования все члены, которые еще содержат расстояние х к моменту вычисления, становятся равными нулю, так что коэффициент соответствующий постоянный член дает множителю соответствующую производную a пункт расчета.

    Для ряда Фурье вместо этого мы начинаем с интегрирования произведения функции и обертонов cos(mωt)or.sin(mωt);m=1,2,3… за один период T фундаментальной частота (м=1)

    ∫ 0Tcos(mωt)f(t)dt= ∫ 0Tcos(mωt)(a02+ ∑n=1∞an cos(nωt)+bn sin(nωt))dt∫ 0Tsin(mωt)f(t)dt= ∫ 0Tsin(mωt)(a02+ ∑n=1∞an cos(nωt)+bn sin(nωt))dt

    Сначала это выглядит немного сложно; однако оказывается, что интеграл над константой, т.е. первый член перед символом суммы почти всегда обращается в нуль, так как интеграл по периоду косинуса или синуса равен нулю. Только для m=0 получается вклад, поскольку имеем cos0=1= константа Поэтому применяется следующее:

    а02=1Т∫ 0Tf(t)dt.

    Кроме того, интеграл по произведению обертона м и секунда обертон сущ. ноль, если м и n не равный. Это также применимо, когда функция косинуса и синуса умножается, потому что функции синуса нечетны, а функция косинуса четна относительно х=0. Поэтому нам осталось только с интегралами cos2nx или sin2nx который обе равны T/2. Таким образом, коэффициенты легко выписываются, но для этого требуется определение интегралов, что требует численных расчетов.

    ан=2T∫ cos(nωt)f(t)dt;bn=2T∫ sin(nωt)f(t)dt

    Моделирование на рис. 5.15 визуализирует эти обстоятельства, которые упрощают расчет коэффициентов Фурье. Из поля выбора продукт периодических функций интересующего нас общего вида есть выбрано: cos(mx)(acos(nx)+bsin(nx)). Красная кривая представляет произведение в cosmx и регулируемый обертон acosnx+bsinnx, на рисунке имеем m=10 и n=8. синяя кривая показывает интеграл, конечное значение которого (определенный интеграл за один период f(t)) обращается в нуль для m≠n. За m=n получаем, когда интегрирование по acosmxcosmx результат aπ, а интеграл над смешанным термином bcosmxsinmx исчезает. Интеграция запускается выбором соответствующей опции коробка.

    С слайдами параметры a и б и целые числа м и н может быть выбранным. Функция обведена красным. После активации поля под названием Интеграл интегральная функция синего цвета вычисляется по периоду основного колебания от 0 до 2π. Окончательное значение представляет собой интересующий нас определенный интеграл.

    В качестве первого шага убеждаемся, что интегралы по синусу и косинус обращается в нуль и что сложение функций синуса и косинуса приводит к сдвинутая по фазе функция синуса или косинуса, интеграл которой также равен нулю. вычисление интеграла от произведения определенной выше функции на обертона изначально неизвестного порядка показывает, что действительно все вклады исчезают, за исключением того, где обертоны идентичны, а функция тип (синус или косинус) одинаков. Понятно, что симметрия различные функции относительно середины периода на ось x является причиной этого конкретного результата. Таким образом, мы имеем.

    ∫ 0Tcos(mωt)dt=0;∫ 0Tcos(mωt)sin(nωt)dt=0;

    ∫ 0Tcos(mωt)cos(nωt)dt=0 для m≠nT∕2form=n

    Это свойство функций синуса и косинуса означает, что они являются примером ортогональная система функций. Две функции называются ортогональными, если применяется следующее:

    ∫ 0Tf1(t)f2(t)dt=0forf1(t)≠f2(t)


    Рисунок 5.15: Симуляция визуализирует ортогональность тригонометрической функции.


    На страницах описания симуляции более подробные инструкции и подсказки для предусмотрены эксперименты. После открытия симуляции вы выбираете тип функции и нажмите клавишу ввода. Процесс интеграции анимирован для вас легче увидеть разницу между интегралами при изменении функции.

    5.10.3 Визуализация расчета коэффициентов и спектра

    Моделирование на рис.5. 16 визуализирует вычисление коэффициентов Фурье для основной тон и первые девять обертонов для следующего типичного периодического издания функции: пилообразная, прямоугольная волна, прямоугольный импульс и импульс Гаусса. К этому концу произведение функций под знаком интеграла определяется и рисуется красным цветом а определенный интеграл показан синим цветом. Окончательное значение интеграла равно, за исключением фактор π который был подавлен, чтобы получить более легко читаемые значения, равные коэффициенту выбранный порядок. Функции предоставляются с тремя параметрами группа с этим контролировать амплитуду, точку симметрии и ширину импульса. От моделирования спектры показанных функций могут быть получены в численном и экспериментальным способом.


    Рисунок 5.16: Расчет коэффициентов Фурье для выбора функций F(t)для колебание зуба пилы.


    Интерактивный рисунок моделирования показывает ситуацию для синуса коэффициенты десятого порядка симметричного зуба пилы. Моделирование начинается с выбор функции и нажатие на клавишу ввода. Страницы с описанием и инструкции для экспериментов содержат дополнительные подробности.

    5.10.4 Примеры разложения Фурье

    В следующих интерактивных примерах (рис. 5.17 — рис. 5.19) вычисление коэффициенты происходят в фоновом режиме. В окне функция показан красным, а частичная сумма желаемого порядка показана синим цветом. Окно функций является интерактивным, так что можно использовать гораздо больше функций. введены и несколько предложены в описании. В текстовом окне порядок анализа можно регулировать; с ползунком порядок приближения н быть используется для частичной суммы. Моделирование позволяет использовать очень высокие заказы.

    Вычисление разложения Фурье n-й заказ следует сразу после входа в функцию. Диаграмма выходит за рамки интеграции область 2π чтобы увидеть периодическое продолжение в обе стороны.

    На рис.5.17 разложение Фурье порядка 43 это показано как приближение для симметричного и периодического прямоугольного импульса. Для прямоугольная волна очень четко распознает типичный выброс на разрывах, которая не обращается в нуль даже для очень высоких порядков.


    Рисунок 5.17: Периодический прямоугольный импульс (красный) и его приближение Фурье (синий) 43-го порядка. Расчетный заказ н можно выбрать.


    На рис.5.18 с использованием того же моделирования показано приближение 17-го порядка для пилообразных колебаний, которые были модулированы нелинейным образом с синусоидальная функция высокой частоты.


    Рисунок 5.18: Периодическая пилообразная форма, модулированная из середины периода с помощью высокочастотной синусоидальной функции (красный) и приближения Фурье 17-го порядок (синий). Частоту модуляции можно выбрать с помощью ползунка. Похожий сложные формы волны используются в синтезаторах для получения интересных звуков.


    Во втором окне симуляции (рис. 5.19) показан спектр. Это можно изменить между синусом (а)-, косинус (bn) и спектр мощности (sn2+bn2). На этом рисунке показан спектр модулированной пилообразной формы, богатый обертонами. и имеет ярко выраженную форманту на шестом и седьмом обертоне. В акустике форманты определяются как ограниченные области обертонов с большой амплитудой; Oни в значительной степени определяют качество тона.

    .


    Рисунок 5.19: частотный спектр для Фурье-разложения модулированная пилообразная форма на рис.5.18. По оси абсцисс показан порядок н обертона (основной тон n=1), по ординате можно выбирать между отображением отдельных коэффициентов или полная мощность в заданном порядке.


    Описание симуляции содержит дальнейшие инструкции.

    5.10.5 Комплексный ряд Фурье

    В пространстве комплексных чисел ряд Фурье можно сформулировать очень просто. элегантный способ:

    f(t)= ∑n=-∞∞cneinωtcn=1T ∫ 0Tf(t)einωtdt.

    Связь с реальным представлением получается путем переупорядочения суммы и объединения, начиная с n=1 термины с -n и н. Принимая во внимание cos(-x)=cos(x);sin(-x)=-sin(x) мы получаем

    f(t)= ∑p=-∞∞cneinωt= ∑p=-∞∞cn(cosnωt+isinnωt)=c0+(c1+c-1)cosωt+i(c1-c-1)sinωt+…f (t)=c0+ ∑p=1∞(cn+c-n)(cosnωt+i(cn-c-n)sinnωt)

    В качестве связи между вещественными и комплексными коэффициентами получаем

    a0=2c0;an=cn+c-n;bn=i(cn-c-n).

    Комплексный состав особенно используется в электротехнике. Он имеет Преимущество в том, что вычисления с экспонентами в целом проще и удобнее. прозрачными, чем те, у которых есть тригонометрическая функция.

    Для быстрого численного расчета компонентов ряда Фурье a был разработан специальный алгоритм, известный как FFT (Fast Fourier Трансформация). БПФ

    5.10.6 Численное решение уравнений и итерационные методы

    В математике и физике часто требуется определить значения переменная, для которой функция, зависящая от этой переменной, имеет определенное значение Можно такая же проблема

    Итерация в том, что касается вычислений, заключается в том, чтобы найти значение переменная, у которой две функции одной переменной имеют одинаковое значение. Один решает эти задачи через поиск нулей функции.

    f мы определяем y1=f(x);y2=g(x), для которого x равен y1=C? ответ: f(x)-C=0, для которого x равен y1=y2? ответ: h(x)≡ f(x)-g(x)=0

    Аналитическое решение для нахождения нулей функции может быть найдено только для очень простые функции, поэтому это исключение. Поэтому нужен числовой метод решения, который предпочтительно работает для всех функций и всех параметров ценности.

    Это достигается с помощью итерационных методов, которые представляют вопрос в обратном порядке. Сначала берется значение переменной, которое, вероятно, меньше, чем оценивает первый нуль в интересующем интервале и вычисляет абсолютное значение значения функции и ее знака. Затем увеличивают переменную на заданный интервал (конечно, можно начать справа и уменьшить шаг переменной на шаг). Является ли новое абсолютное значение для того же знака одним переходом к следующему точка. Если знак меняется, очевидно, что вы пересекли ноль. Теперь направление движения инвертируется, а ширина шага умножается на коэффициент <1. Таким образом находятся ящики уменьшающегося размера, содержащие нуль до тех пор, пока отклонение значение функции от нуля становится меньше заданного допуска. затем процесс продолжается в исходном направлении, пока не исчезнут все нули. были найдены или до определенного порога для значения переменной или сама функция была превышена, и, таким образом, человек находится за пределами области интерес.

    Для этого итерационного процесса доступны готовые алгоритмы в стандартном числовые компьютерные коды, которые включают дальнейшие уточнения. Таким образом, можно, для например, варьировать ширину интервалов итераций так, чтобы характер учитывается функция. Например, в методе Ньютона используется его наклоните первую производную, чтобы скорректировать эти интервалы. Учитывая скорость сегодняшнего компьютеры эти усовершенствования не играют роли для простых задач. Следующее интерактивный пример на рис. 5.17 определяет нули функции, которые могут быть вошел по желанию. Эта функция задается как многочлен четвертой степени с иррациональные корни.

    Последовательность показывает ход выполнения очень простого итерационного алгоритма. скорость можно регулировать. Начальную точку итерации (пурпурный) можно перетаскивать с мышью. Итерация продолжается с постоянной шириной шага к большему x-значения пока знак функции не изменится. Начальное значение сбрасывается до последнего значение до смены знака, а ширина шага уменьшается в 10 и прогресс к большим значениям x возобновляется. Это повторяется до тех пор, пока не произойдет отклонение y-значение от нуля падает ниже заданного допуска. В моделировании можно выбрать, будет ли останавливается после достижения определенной точности, или все нули в переменном интервале определяются последовательно. В одном вычислении пурпурная точка перескакивает на расчетное значение. в то время как синяя точка показывает значение первой итерации при определении несколько нулей.

    Для того, чтобы иметь возможность следовать прогрессивной итерации также для высокой точности уже достигнуто хорошо, часть окна подробно показана в увеличительном стекле, и шкала подстраивается под возрастающую точность.

    Из окна масштабирования на Рис. 5.20 видно, что кривая всегда почти linear закрывает корень кривой. Regula Falsi использует в качестве следующего значения итерации для х пересечение секущей, образованной двумя предыдущими точками итерации, с ось х. Поэтому он быстро приводит к окончательному решению. Однако мы выбрали константу ширину шага, чтобы было легче наблюдать за процессом.


    Рисунок 5.20: Анимированное итеративное вычисление нулей функции, многочлен четвертой степени на рисунке. Левое окно показывает весь расчетный интервал, правый участок, шкала которого соответствует достигнутому разрешению. Показана последняя точка итерации синего цвета в обоих окнах, а три предшественника показаны в красный цвет в «зазеркалье». на картинке для возврата после деления интервала на 10. Пурпурная точка является начальной точкой итерация. Его можно нарисовать мышкой. Желаемая точность дельта , количество временных шагов в секунду (скорость) и диапазон абсцисс хмакс можно выбрать. В числовых полях координаты текущей точки итерации х, у и начальная точка х0, у0 показаны итерации. В окне формулы можно вводить любые функции нули которого необходимо вычислить.


    Более подробную информацию и советы по проведению экспериментов можно найти на страницах описания симуляция.

    Конец главы 5

    11.3: Серия Фурье II — Mathematics LibreTexts

    1. Последнее обновление
    2. Сохранить как PDF
  • Идентификатор страницы
    9462
    • Уильям Ф. Тренч
    • Университет Тринити

    В этом разделе мы обсуждаем разложения Фурье по собственным функциям задач 1-4 для раздела 11.1.

    Косинусоидальный ряд Фурье

    Из Упражнение 11.1.20 собственные функции

    \[1,\, \cos{\pi x\over L}, \, \cos{2\pi x\over L},\ точки, \, \cos{n\pi x\over L},\dots\nonumber \]

    9Lf(x)\cos{n\pi x\over L}\,dx,\quad n=1,2,3,\dots. \nonumber \]

    Сравнение этого определения с теоремой 11.2.6a показывает, что функция Фурье Косинусный ряд \(f\) на \([0,L]\) — это ряд Фурье функции

    \[f_{1}(x)=\left\{\begin{array}{cc}{ f(-x),}&{-L

    получается расширением \(f\) на \([-L,L]\) как четная функция (рис. 11.3.1). ).

    Рисунок 11.3.1

    Применение теоремы 11.2.4 к \(f_1\) дает следующую теорему. 9Lf(x)\cos{n\pi x\over L}\,dx,\quad n=1,2,3,\dots,\nonumber \]

    сходится для всех \(x\) в \([0,L];\), причем

    \[C(x)=\left\{\begin{array}{cl}{f(0+),}&{\text{if}x=0}\\[5pt]{f(x), } & {\ text {если} 0

    Пример 11.3.1

    9{2}}\cos\frac{(2n-1)\pi x}{L}\номер\]

    Теорема 11.3.1 подразумевает, что

    \[C(x)=x,\quad 0\le x\le L. \не число \]

    Синусоидальный ряд Фурье

    Из Упражнение 11.1.19 , собственные функции

    \[\sin{\pi x\over L}, \, \sin{2\pi x\over L},\dots, \, \sin{n\pi x\over L},\dots\nonumber \]

    краевой задачи

    \[y»+\lambda y=0,\quad y(0)=0,\quad y(L)=0\nonumber \]

    (задача 1) ортогональны на \([0,L]\). Если \(f\) интегрируема на \([0,L]\), то разложение Фурье \(f\) по этим функциям называется 9Lf(x)\sin{n\pi x\over L}\,dx,\quad n=1,2,3,\dots.\nonumber \]

    Сравнение этого определения с теоремой 11.2.6b показывает, что функция Фурье ряд синусов \(f\) на \([0,L]\) — это ряд Фурье функции

    \[f_{2}(x)=\left\{\begin{array}{cc}{ -f(-x),}&{-L

    , полученное расширением \(f\) над \([-L,L]\) как нечетная функция (рис. 11.3.2). ).

    Рисунок 11.3.2

    Применение теоремы 11.2.4 к \(f_2\) дает следующую теорему. 9Lf(x)\sin{n\pi x\over L}\,dx,\nonumber \]

    сходится для всех \(x\) в \([0,L];\), причем

    \[S(x)=\left\{\begin{array}{cl}{0}&{\text{if}x=0}\\[5pt]{f(x),}&{\text {если}0 \номер\]

    Пример 11.3.2

    Найдите ряд Фурье по синусам \(f(x)=x\) на \([0,L]\). 9n\over n} \sin{n\pi x\over L}.\nonumber \]

    Теорема 11.3.2 следует, что

    \[S(x)= \left\{\begin{array}{cl} x,&0\le x< L,\\0,& x=L. \end{array}\right.\nonumber \]

    Смешанный ряд косинусов Фурье

    Из Упражнение 11.1.22 собственные функции

    \[\cos{\pi x\over 2L}, \, \cos{ 3\pi x\over 2L},\dots, \, \cos{(2n-1)\pi x\over 2L},\dots\nonumber \]

    краевой задачи

    \[\label{ eq:11.3.2} y»+\lambda y=0,\quad y'(0)=0,\quad y(L)=0\] 9Lf(x)\cos{(2n-1)\pi x\over2L}\,dx.\nonumber \]

    Мы будем называть это разложение смешанным косинусным рядом Фурье функции \(f\) на \( [0,L]\), потому что граничные условия (Уравнение \ref{eq:11.3.2}) «смешанные» в том смысле, что они требуют, чтобы \(y\) было равно нулю в одной граничной точке и \(y’ \) быть равным нулю на другом. Напротив, «обычный» ряд косинусов Фурье связан с ( Уравнение \ref{eq:11. 3.1}), где граничные условия требуют, чтобы \(y’\) было равно нулю в обеих конечных точках.

    Можно показать ( Упражнение 11.3.57 ), что смешанный ряд косинусов Фурье \(f\) на \([0,L]\) является просто ограничением на \([0,L]\) ряда косинусов Фурье

    \[f_3(x)=\left\{\begin{array}{cl} f(x),&0\le x\le L,\\-f(2L-x), &L< x\le 2L \end{array}\right.\nonumber \]

    on \([0,2L]\) (рис. 11.3.3) ).

    Рисунок 11.3.3

    Применение теоремы 11.3.1 с заменой \(f\) на \(f_3\) и \(L\) на \(2L\) получается следующая теорема. 9Lf(x)\cos{(2n-1)\pi x\over2L}\,dx,\nonumber \]

    сходится для всех \(x\) в \([0,L];\), причем

    \[C_{M}(x)=\left\{\begin{array}{cl}{f(0+),}&{\text{if}x=0}\\[5pt]{f( x),} & {\ text {если} 0

    Пример 11.3.3

    Найдите смешанный ряд косинусов Фурье \(f(x)=x-L\) на \([0,L]\). 92} \cos{(2n-1)\pi x\over2L}.\nonumber \]

    Теорема 11.3.3 подразумевает, что

    \[C_M(x)= x-L,\quad 0\le x\le L.\nonumber \]

    Смешанный ряд синусов Фурье

    Из Упражнение 11.1.21 собственные функции

    \[\ sin{\pi x\over 2L}, \, \sin{3\pi x\over 2L},\dots, \, \sin{(2n-1)\pi x\over 2L},\dots\nonumber \ ]

    краевой задачи

    \[y»+\lambda y=0,\quad y(0)=0,\quad y'(L)=0\nonnumber \]

    (задача 3) ортогональны на \([0,L]\). Если \(f\) интегрируема на \([0,L]\), то Фурье-разложение \(f\) по этим функциям равно 9Lf(x)\sin{(2n-1)\pi x\over2L}\,dx.\nonumber \]

    Мы назовем это разложение смешанным рядом синусов Фурье для \(f\) на \( [0,L]\).

    Можно показать ( Упражнение 11.3.58 ), что смешанный ряд Фурье по синусам \(f\) на \([0,L]\) является просто ограничением на \([0,L]\) ряда синусов Фурье

    \[f_4(x)= \left\{\begin{array}{cl} f(x),&0\le x\le L,\\f(2L-x),&L < x\le 2L, \end{array}\right. \nonumber \]

    on \([0,2L]\) (рис. 11.3.4 ). 9Lf(x)\sin{(2n-1)\pi x\over2L}\,dx,\nonumber \]

    сходится для всех \(x\) в \([0,L];\), причем

    \[S{M}(x)=\left\{\begin{array}{cl}{0,}&{\text{if}x=0}\\[5pt]{f(x),} &{\ text{if}0

    Пример 11.3.4

    Найдите смешанный ряд Фурье по синусам \(f(x)=x\) на \([0,L]\). 92} \sin{(2n-1)\pi x\over2L}.\nonumber \]

    Теорема 11.3.4 следует, что

    \[S_M(x)=x,\quad 0\le x\le L.\nonumber \]

    Полезное наблюдение

    В приложениях, включающих разложения по собственным функциям задач 1-4, разлагаемые функции часто являются полиномами, удовлетворяющими граничным условиям рассматриваемой задачи. В этом случае следующая теорема предлагает эффективный способ получения коэффициентов в разложении.

    9L f»(x)\sin{(2n-1)\pi x\over2L} \,dx.

    Добавить комментарий

    Ваш адрес email не будет опубликован. Обязательные поля помечены *